You are on page 1of 41

1) Un estudio es realizado para evaluar una B) Hemograma completo

nueva prueba serológica para HIV. 1000 C) Electrocardiograma


personas fueron seleccionadas al razón de la D) Glucosa en ayunas y lípidos
población, se evaluaron utilizando la nueva E) Pruebas de función hepática
prueba y nuevamente utilizando la prueba
estándar (por ejemplo el Western Blot) para 3) Un hombre de 23 años que recientemente
determinar su real estatus infeccioso. Los se sometió a trasplante de médula ósea por
hallazgos fueron los siguientes leucemia mieloide aguda se queja de fiebre,
diarrea severa, prurito e ictericia. La biopsia
hepática muestra marcada disminución de la
cantidad de conductos biliares. ¿Cuál de las
siguietes es caracterizada por ese mismo
patrón de lesión hepática?
A) Toxicidad por acetaminofén
Cuál es la sensibilidad de esta nueva prueba
B) Enfermedad hepática alcohólica
serológica?
C) Colangitis esclerosante primaria
A) 140/180
D) Cirrosis biliar primaria
B) 140/200
E) Nutrición parenteral total
C) 60/820
D) 60/200
4) Un hombre de 65 años acude a urgencias
E) 180/100
con dolor torácico sub esternal, dificultad
respiratoria, diaforesis que inició
2) Una joven de 28 años es evaluada por la
repentinamente hace 40 minutos. Desde
aparición reciente de pensamiento
que inició el dolor, el paciente ha vomitado
paranoide. Por los últimos 4 meses, ella ha
en 2 ocasiones. El dolor se irradia al brazo
tenido pensamientos que sus compañeros de
izquierdo y no remite con nitroglicerina
trabajo han estado espiándola y hablando a
sublingual. El EKG muestra elevación del ST
sus espaldas. Ella tiene miedo de ir a
de 2mm en las derivadas anteriores. Al
trabajar y se preocupa por perder su empleo.
examen físico el paciente tiene temperatura
NO ha tenido alucinaciones o depresión y no
de 36.9°C, presión arterial de 110/70mmHg,
hay antecedentes médicos relevantes. Su
pulso de 60/min y respiración de 32/min con
padre tiene enfermedad arterial coronaria e
una saturación de oxígeno del 90% utilizando
hipertensión y su madre tiene antecedente
oxígeno por cánula nasal a 4L7Min. El
de depresión. La paciente tiene antecedente
examen físico muestra un S1 y S2 apagados
de fumar 9 paquetes / año de cigarrillos y
con presencia de un S3. En los pulmones hay
bebe alcohol solo de forma social. Al
crépitos bibasales que se extienden hacia la
examen físico no hay hallazgos patológicos
mitad de los campos pulmonares de manera
relevantes. Se le inicia olanzapina. ¿Cuál de
bilateral. ¿Cuál de los siguientes es el mejor
los siguientes pruebas debe ser la más
próximo paso en el manejo del paciente?
importante a realizar luego de 3 meses de
A) Metoprolol
estar en terapia con olanzapina?
B) Digoxina
A) Nitrógeno de urea y creatinina
C) Furosemida
D) Espironolactona paquetes de cigarrillos diarios por 30 años.
E) Fluidos IV Bebe 5-6 onzas de alcohol diariamente.
Consume mucha comida chatarra y se
5) Un hombre de 64 años es sometido a una ejercita de forma infrecuente. Ambos padre
resección colónica de urgencia por colitis y madre tienen diabetes e hipertensión. Su
isquémica extensiva. La cirugía fue sin hermano mayor murió de infarto miocárdico
complicaciones. El paciente recibió hace 2 años. Sus medicamentos incluyen
piperacilina – tazobactam por los últimos 5 gliburida, lisinopril, atenolol, simvastatina y
días. Ha estado en nada por boca desde famotidina. Su temperatura es 37.8°C,
hace 5 días. Tiene antecedente de presión arterial de 160/90 mmHg, pulso de
alcoholismo significativo. En el día post 88/min y respiratorio de 16/min. Al examen,
operatorio 6 desarrolla sangrado por el sitio el dedo gordo derecho impresiona
de venopunción. Su temperatura es de marcadamente hinchazo, rojo y caliente al
36.7°C, presión arterial es de 121/76mmHg, tacto. A la aspiración del líquido revela
pulso de 80/min y respiración de 16/min. Los cristales en aguja birefringente negativo.
resultados de laboratorio son: Adicional a la ya prescrita indometacina oral,
Hemoglobina: 11.5g/dl; VCM: 88 fl. cuál otra intervención sería la más
Plaquetas: 160,000/mm3, Glóbulos blancos: importante para prevenir futuros ataques de
7,500/mm3,, Neutrófilos: 68% Bandas : 1%. gota?
Eosinófilos: 1%, Linfocitos: 24%, A) Cesación tabaquismo
Monocitos: 6%, Tiempo de protrombina: 20 B) Cesación de alcoholismo
seg (INR = 1.9), Tiempo de trombolastina C) Omitir lisinopril
parcial activada: 45 seg. No hay D) Omitir simvastatina
antecedentes familiares de importancia. E) Omitir gliburida
¿Cuál de las siguientes es la causa más
probable de esta condición? 7) Un hombre de 23 años de edad,
A) Púrpura trombocitopénica trombótica previamente sano se presenta al servicio de
B) Síndrome urémico hemolítico urgencias tras sufrir una herida de bala en el
C) Púrpura trombocitopénica idiopática pecho izquierdo. La herida de entrada está 3
D) Deficiencia vitamínica cm debajo del pezón y la herida de salida es
E) Coagulación intravascular diseminada justo debajo de la escápula. Un tubo torácico
se coloca y drena 400 ml de sangre y
6) Un hombre caucásico de 53 años acude a continúa drenando 50-75 ml / h durante la
urgencias a las 2AM por dolor severo en su reanimación inicial. La presión arterial inicial
dedo gordo derecho en las últimas 2 horas. de 70/0 mmHg responde a 2 litros de
El se queja que el dolor inició cristaloides y ahora está 100/70 mmHg. El
repentinamente, es sordo, doloroso y examen abdominal sin nada especial. La
empeora en el tiempo a palpitación severa, radiografía de tórax revela un pulmón
no aliviado con acetaminofén. Sus otros reexpandido y no hay aire libre bajo el
problemas médicos incluyen hipertensión, diafragma. El siguiente paso en el manejo
diabetes mellitus tipo 2, hipercolesterolemia debe ser:
y enfermedad úlcera péptica. Ha fumado 2 A) Admisión y observación
B) El lavado peritoneal su piel, polidipsia, poliuria y disminución del
C) Toracotomía exploratoria libido. Ha tenido buena salud hasta ahora, y
D) Laparotomía exploratoria su revisión médica hace 1 año fue normal.
E) La exploración de la herida local Los resultados de laboratorio son: Ferritina
sérica 1100 ng/ml; Saturación de transferrina
8) Una mujer de 59 años, G4P4, sse presenta 55% (normal <45%); Glucosa plasmática: 170
a su oficina por de perder orina cuando tose, mg/dl; Alanino aminotransferasa 120 UI/L;
estornuda, o que realicen determinados Aspartato aminotransferasa: 100 UI/L; Si no
tipos extenuantes de actividad física. El recibe tratamiento, este paciente está en
problema se ha vuelto cada vez peor en el riesgo de desarrollar cuál de las siguientes
último año, hasta el punto donde el paciente complicaciones?
encuentra sus actividades de la vida diaria A) Leucemia mieloide aguda
comprometidas por miedo a la vergüenza. B) Carcinoma adrenal
Ella niega cualquier otra síntomas urinarios C) Tumor cerebral
tales como urgencia, frecuencia, o D) Carcinoma hepatocelular
hematuria. Niega cualquier problema con sus E) Cáncer pancreático
movimientos intestinales. Sus cirugías
previas incluyen una amigdalectomía y 10) Un hombre de 43 años se queja de
apendicectomía. Tiene Antecedentes de frecuente sensación de ardor o quemazón
Diabetes tipo 2, con glicemias bien epigástrica que no se alivia con los
controlados con antidiabéticos orales . No antiácidos. SE presenta típicamente cuando
tiene antecedentes de problemas debe levantar objetos pesados en el trabajo
ginecológicos en el pasado. Tuvo 4 hijos por y demora 10 a 15 minutos en aliviarse. El
parto vaginales espontáneos; sus pesos niega asociación con dolor en cuello o
oscilado entre 8 y 9 libras. Actualmente es brazos, tos, disnea o dificultad para tragar.
sexualmente activa con su pareja de 25 años. Sus antecedentes incluyen lupus eritematoso
Hace 4 años con menopausia y nunca ha sistémico diagnosticado hace 5 años, para lo
tomado terapia de reemplazo hormonal. Su cual toma baja dosis de prednisona diaria. Al
altura es de 167cm., peso de 190lbs Su examen físico la presión arterial es de 140/90
presión arterial es de 130/80 mmHg. mmHg, pulso de 80/min, regular. No hay
Basados en los antecedentes de la paciente, hallazgos patológicos al examen de corazón,
¿cuál es el diagnóstico más probable? pulmones y abdomen. El EKG es normal.
A) Incontinencia por rebosamiento ¿Cuál de los siguientes es el mejor próximo
B) Incontinencia de esfuerzo paso en el manejo de este paciente?
C) Infección del tracto urinario A) Ecocardiograma
D) Inestabilidad del detrusor B) Angiografía coronaria
E) fístula vesicovaginal C) Prueba de esfuerzo
D) Prueba de perfusión miocárdica
9) Un hombre de 45 años acude a su médico E) Estudios de motilidad esofágica
quejándose de 2 meses de pérdida de peso, F) Endoscopia del tubo digestivo alto
debilidad general. Durante este periodo, ha 11) Una mujer de 46 años sufre de Lupus
notado un aumento en la pigmentación de eritematoso sistémico desarrolla hematuria y
proteinuria. Ella fue diagnosticada con Lupus paciente?
hace 4 años atrás y ha sido tratado con A) Cubrir el ojo afectado
hidroxicloroquina y prednisona. La biopsia B) Cubrir el ojo normal
renal se realiza y reporta glomerulonefritis C) Referir a oftalmología
proliferativa difusa. Ella inicia ciclofosfamida D) Consejería
y la hidroxicloroquina se detiene. Este nuevo E) Vigilancia y seguimiento
régimen pone a la paciente en un riesgo más
elevado de cuál de las siguientes 14) Un niño de 15 años de edad es detenido
condiciones? por disparar el propietario del supermercado
A) Disfunción coclear que trató de robar. Él ha estado en 'custodia
B) Neuritis óptica varias veces por una variedad de delitos
C) Carcinoma de vejiga contra la propiedad, la posesión de
D) Neuropatía periférica sustancias ilegales, y el asalto. Él está alegre
E) Vasoespasmo digital y despreocupado durante la detención, más
preocupado por no perder su chaqueta de
12) Un hombre de 50 años acude al médico cuero que sobre el destino del hombre que
por historia de 2 semanas de rash ha lesionado. Elija el diagnóstico más
pruriginoso en la superficie extensora de apropiado:
piernas y brazos. El paciente niega historia A) Trastorno de oposición desafiante
personal o familiar de desórdenes alérgicos. B) Desorden de personalidad antisocial
Las características del rash son: eritematoso, C) Desorden de personalidad narcisista
costroso, placas en forma de moneda. D) Desorden de conducta
Raspado de las lesiones tratado con E) Abuso de sustancias
hidroxido de potasio (KOH) no revela esporas
o hifas al examen de microscopia. El resto 15) mujer posmenopáusica está bajo
del examen físico es normal. Cuál es el evaluación por incontinencia fecal. Ella no
diagnóstico más probable? tiene otros problemas médicos
A) Dermatitis alérgica de contacto diagnosticados. Vive sola y es autosuficiente,
B) Eczema Numular orientada, y una excelente historiadora.
C) Psoriasis Examen físico
D) Deficiencia vitamínica es completamente normal. ¿Cuál de los
E) Lupus eritematoso sistémico siguientes es la causa más probable de la
condición de esta paciente?
13) Una niña de 2 meses de edad, nacida a A) Prolapso rectal
término es llevada a su control de B) Diabetes
crecimiento. No hubo problemas antenatales C) Trauma obstétrico
ni en el parto. Su desarrollo, peso y talla han D) Senilidad
estado dentro de los rangos normales para E) Consumo excesivo de cafeína
su edad. La evaluación oftalmoscópica
revela un reflejo blanco en el ojo derecho. El 16) Una mujer de 45 años acude con quejas
resto del examen es normal. ¿Cuál es el más de constipación y dolor abdominal de 2
apropiado paso a seguir en el manejo de este semanas. Ella refiere polaquiuria y constante
sed. Sus antecedentes médicos destacan Calcio: 11.7 mg/dl
obesidad. Ella refiere que ha tratado muy Albúmina: 3.7 g/dl ¿Cuál de las siguientes es
duro de perder peso, y en los pasados 6 la causa más probable de la hipertensión de
meses ha intentado varios tipos de dietas. este paciente?
Ella le comenta que se suplementa con A) Enfermedad del parénquima renal
numerosos tipos de vitaminas y minerales B) Estenosis vascular renal
compradas sin receta médica. Ella ha C) Coartación de la aorta
logrado la pérdida de 20 lbs durante este D) Hipotiroidismo
tiempo. Entre sus antecedentes también hay E) Enfermedad paratiroidea
fibrilación atrial desde hace 4 años, para la
cual toma diltiazem. Al examen físico su 18) Se realiza un estudio para evaluar la
temperatura es 36.8°C, presión arterial de relación entre etnicidad y enfermedad renal
120/70 mmHg, pulso de 90/min y respiración terminal. Dos grupos de patólogos
en 13/min. Su membrana mucosa está seca, independientemente estudiaron los
su abdomen es blando, no doloroso sin especímenes de 1,000 biopsias renales. El
defensa ni rebote ni rigidez. Los sonidos primer grupo de patólogos estaba al tanto de
intestinales están presentes. El urinálisis la raza del paciente de quien venia la biopsia,
está dentro de los límites normales Cuál de mientras que el segundo grupo desconocía la
los siguientes es más probable responsable raza del paciente. El primer grupo reportó
de los síntomas actuales de la paciente “nefropatía hipertensiva” con mayor
A) Diltiazem frecuencia para pacientes de raza negra que
B) Cetoacidosis diabética el segundo grupo. ¿Cuál es el más probable
C) Sobredosis de vitamina D tipo de error, entre los siguientes, que
D) Insuficiencia adrenal estuvo presente en este estudio?
E) Sobredosis de vitamina A A) Confusión
F) Malabsorción B) sesgo por el interrogado
C) Sesgo por recuerdo
17) Una mujer de 42 años es evaluada por D) Sesgo de selección
depresión, cambios en el humor y pobre E) Sesgo del observador
calidad de sueño. También se queja de
cefaleas moderada y debilidad muscular. Ha 19) Un hombre de 45 años es llevado a
tenido 2 visitas a urgencias por cálculo urgencias luego de un accidente de carro. El
renales el año pasado. No usa drogas ilícitas. es incapaz de dar la historia. Al examen físico
Su presión arterial es 160/105mmHg y su hay sangre en el meato uretral y un
pulso está en 85/min. Los laboratorios hematoma escrotal derecho. Su temperatura
obtenidos son: es de 37°C, presión arterial de 110/75, pulso
Sodio: 140 mEq/L de 94/min y respiratoria en 18/min. El
Potasio: 3.6mEq/L examen físico muestra una próstata
Cloro: 101 mEq/L cabalgada sin otro signo de trauma. ¿Cuál de
Bicarbonato: 24mEq/L las siguientes es el más apropiado paso a
Creatinina: 0.9 mg/dl seguir en el manejo del paciente?
Glucosa. 98 mg/dl A) Cirugía inmediata para reparar la uretra
B) Cateterización con sonda foley es de 7.1 mg/dl. ¿Cuál de las siguientes es la
C) Uretrograma retrógrado causa más probable de la anemia de este
D) Diuréticos para aumentar el gasto urinario paciente?
E) Cistograma retrógrado con placas post A) Deficiencia de hierro
miccionales. B) Deficiencia de folato
C) Deficiencia de cobalamina
20) Un niño de 8 años, previamente sano es D) Deficiencia de glucosa 6 fosfato
llevado a consulta por su madre por deshidrogenasa
múltiples episodios con la mirada perdida. E) Infiltración a médula ósea
Durante estos periodos el niño queda sin F) Hemoglobinopatía
respuesta verbal o al estímulo táctil y
produce movimientos de estar masticando 22) Un hombre de 60 años acude al médico
con los labios. Cada episodio dura algunos para evaluación de dolor lumbar y dificultad
minutos, luego de los cuales queda para caminar de varios meses. El dolor es
confundido por un periodo de tiempo peor cuando está mucho tiempo de pie y se
adicional. No hay historia familiar de ningún alivia al sentarse. También el dolor es menos
tipo de convulsión. Al examen neurológico severo subiendo una colina que bajándola.
sin hallazgos patológicos. Se realiza un EEG En sus antecedentes está la degeneración
antes y después de la hiperventilación el cual articular de ambas rodillas. No consume
se reporta como normal. ¿Cuál de las tabaco, alcohol o drogas. Toma
siguientes es el diagnóstico más probable? acetaminofén cuando es necesario. Su
A) Convulsión parcial compleja temperatura es 37.2°C, presión arterial de
B) Convulsión de ausencia típicia 126/76mmHg. Su abdomen es blando, no
C) Convulsión de ausencia atípica doloroso. No hay dolor espinal a la palpación
D) Epilepsia juvenil mioclónica y la prueba de estirar la pierna no provoca
E) Síndrome de Lennox – Gastaut dolor. Los reflejos están 2+ en ambas
piernas. No hay signo de Babinski. Se palpan
21) Un hombre de raza blanca de 72 años normales e iguales los pulsos pedio dorsal en
acude a su oficina con quejas de fatiga ambos pies. ¿Cuál de los siguientes sería más
severa. El dice que hace 5 meses el era capaz útil en el diagnóstico de la condición actual?
de subir 4 tramos de escalera sin disnea, A) Medida del índice tobillo
pero ahora tiene que descansar después del B) Ultrasonido doppler de las extremidades
segundo. Su apetito ha disminuido, pero inferiores
niega dolor o malestar abdominal o heces C) Resonancia de la columna
oscuras. Al examen físico se encuentran D) Electromiografía y velocidad de
nódulos linfáticos agrandados en las regiones conducción nerviosa
cervical, inguinal y supraclavicular. Un soplo E) Niveles de HLA – B27
sistólico de grado bajo II/VI es audible en el
borde paraesternal izquierdo. El abdomen es 23) Una mujer de 55 años acude al médico
blando, no doloroso. El hígado mide 10cm y por quejarse de 2 años de tos episódica y
el bazo es palpable 4 cm por debajo del producción de esputo amarillo. Su primer
reborde esternal izquierdo. Su hemoglobina episodio duró alrededor de 3 semanas con
congestión del pecho, tos productiva y pulso de 106/min, presión arterial de
esputo purulento con disnea. Ella vio a su 130/80mmHg. Su pantorrilla derecha está
médico de atención primaria en ese edematizada, eritematosa y
momento y la diagnosticó con bronquitis y le extremadamente dolorosa y caliente al tacto
recetó antibióticos. La paciente desde en una región aproximada de 6x3cm. Hay
entonces ha tenido 6 episodios adicionales, una masa dolorosa palpable en la ingle
cada uno con gran cantidad de esputo derecha. No hay crépitos articular ni bullas
amarillo (algunas veces teñidos de sangre), visibles. Sus piel entre los dedos están
disnea y congestión a nivel de senos fisurados y macerados. Los laboratorios
paranasales. Ella mejora en cada ocasión con muestran: Hemoglobina 14.0 g/L; plaquetas
antibióticos. Sin embargo, ella continua 222,000/mm3; glóbulos blancos en
teniendo tos diaria y se ha empeorado en las 14,500/mm3. Neutrófilos 86% Linfocitos
últimas 2 semanas. NO tiene fiebre, 14% ¿Cuál de los siguientes es la causa más
escalofríos, dolor torácica, mascotas en casa probable de la condición actual de la pierna?
ni ha viajado recientemente. Tiene A) Celulitis
antecedente de tabaquismo de 30 B) Trombosis arterial
paquetes/año. Su temperatura es 37.2°C, C) Trombosis venosa profunda
presión arterial de 120/68mmHg, pulso en D) Fascitis necrotizante
80/min y respiratoria en14/min. Su cuello es E) Ruptura de un quíste de Baker
libre de adenopatías. En los pulmones se
escucha roncus difusos con sibilancias y 25) Un hombre de 69 años se somete a
algunos crépitos gruesos en las bases. El cirugía de bypass arteria coronaria con
resto de la evaluación no muestra reemplazo de válvula aórtica. El
anormalidades. La radiografía muesta procedimiento fue exitoso y el paciente es
lineales atelectasias en los campos extubado y pasado de unidad cuidados
pulmonares medios e inferior intensivos a sala intermedia en el día
bilateralmente. postoperatorio 2. Esa noche, se queja de
¿Cuál de los siguientes es el próximo paso debilidad, opresión torácica y dificultad
más apropiado para confirmar el diagnóstico respiratoria. Su presión arterial es 70/30
de esta paciente? mmHg, respiración de 26/min y la frecuencia
A) Broncoscopia con lavado alveolar cardiaca es de 148/min. La auscultación
B) Ecocardiografía pulmonar revela crépitos bibasales. El EKG
C) tomografía torácica de alta resolución se muestra a continuación:
D) Prueba de funciones pulmonares
E) Esputo por Bacilo alcohol ácido resistente

24) Una mujer obesa de 30 años acude a Cuál de las siguientes es el mejor próximo
urgencias por 4 días de dolor, edema y paso en el manejo de este paciente
enrojecimiento progresivo de la pierna A) Amiodarona
derecha. No tiene trauma o picadura de B) Marcapaso transcutáneo
insectos obvias. No consume alcohol, tabaco C) Cardioversión eléctrica
o drogas ilícitas. Su temperatura de 38.7°C,
D) Lidocaína D) Enalapril
E) Digoxina E) Amiodarona

26) Un hombre de 45 años con linfoma de 28) Un hombre de 45 años acude a urgencias
Burkitt es tratado con una combinación de por dolor abdominal persistente. El dolor
quimioterapia y alopurinol. Al 3er día de inició hace más de 12 horas y es
tratamiento, nota que tiene disminución del predominantemente epigástrico. Ha
gasto urinario con aumento de los niveles de vomitado varias veces desde que el dolor
nitrógeno de urea y creatinina. Al paciente inició. El dolor empeora cuando camina o se
se le inicia hidratación intensa. El EKG acuesta en supino. Se mejora cuando se
muestra intervalos QT prolongados. Se sienta o se inclina hacia el frente. Entre sus
sospecha de síndrome de lisis tumoral. Se antecedentes médicos está hiperlipidemia
ordena un panel metabólico completo. ¿Cuál tratada con simvastatina e hipertensión
de los siguientes conjunto de resultados se tratada con lisinopril. Fuma un paquete de
esperarían en este paciente? cigarrillos día. Bebe 46 latas de cerveza por
día y más durante los fines de semana. No
consume drogas ilícitas. La temperatura es
37.8°C, presión arterial 100/70 mmHg, pulso
de 110/min y respiratoria de 20/min. Al
examen abdominal tiene dolor moderado en
el epigastrio sin defensa ni rebote. La
radiografía muestra efusión pleural
27) Un hombre de 56 años acude a su izquierda. ¿Cuál de los siguientes es más
consulta con quejas de aumento progresivo probable que padezca el paciente?
de su fatiga. Refiere dificultad en la A) Colecistitis aguda
concentración y aumento en los olvidos en B) Pancreatitis aguda
las últimas semanas. En la revisión de C) Absceso intra abdominal
síntomas, el paciente refiere que ha ganado D) Isquemia mesentérica
6 libras de peso en los últimos 3 meses de E) Infarto miocárdico
forma no intencional. Sus antecedentes
médicos son hipertensión arterial y 29) Una mujer de 35 años acude a su médico
fibrilación atrial. Al examen físico tiene la de atención primaria que se queja de
presión arterial de 140/90 mmHg, una palpitaciones, pérdida de peso, aumento de
frecuencia cardiaca de 75/min. Sus campos apetito y diarrea por los últimos 2 meses.
pulmonares están claros a la auscultación. Ella niega uso de alcohol y tabaco. Su
No hay edema de tobillos. Su piel es seca. temperatura es 37.1°C, presión arterial es
¿Cuál de las siguientes drogas es la más 135/80 mmHg, pulso es 100/min y
probablemente responsable de las quejas de respiración son 14/min. Al examen físico
este paciente? muestra exoftalmos, edema palpebral,
A) Hidralazina retracción palpebral y aumento difuso no
B) Metoprolol dolora de la glándula tiroides. Los estudios
C) Verapamilo de laboratorio muestran unos niveles bajos
de hormona estimulante de tiroides y T4 y T3 31) Una niña de 2 años es llevada a consulta
libre aumentadas. El paciente es por tos seca y fiebre de bajo grado. En los
diagnosticado con enfermedad de graves. Se últimos 2 días ha estado muy irritable y con
discuten varias opciones de tratamiento, y dificultad para alimentarla. Tiene una nariz
ella elige el tratamiento a largo plazo con congestionada. Al examen hay taquicardia y
metimazol. ¿Cuál de las siguientes una sibilancias significativas. La evaluación
condiciones tiene riesgo el paciente de de la secreción nasal confirma el diagnóstico
desarrollar debido a la medicación? de infección por virus respiratorio sincitial.
A) Agranulocitosis En el futuro, ¿Cuál de las siguientes tendrá
B) Hipocalcemia un ligero riesgo de desarrollar la niña?
C) Pancreatitis A) Neumonía
D) Hipotiroidismo permanente B) Absceso pulmonar
E) Parálisis del nervio laríngeo recurrente C) Fibrosis cística
D) Asma
30) Una mujer de 35 años acude al médico E) aspergilosis
por dolor y rigidez de las articulaciones de la
muñeca y manos por los últimos meses. Su 32) Un hombre de 49 años con diabetes
rigidez matutina dura más de una hora. acude a su oficina por un examen de rutina
También se queja de hinchazón en las de salud. Le solicita que le haga una prueba
articulaciones. Sus antecedentes médicos de lípidos y usted obtiene la muestra para un
solo son significativos por un episodio similar análisis de perfil lipídico en ayunas. Las guías
el año pasado, el cual resolvió con “Adult Treatment Panel 3 (ATP3)” para
ibuprofeno comprado sin receta. Al evaluar hipercolesterolemia dependen de cuál de los
la articulación muestra su enrojecimiento siguientes niveles de lípidos para
moderado, calor, edema y dolor en las tratamiento?
articulaciones interfalángicas proximales y A) HDL -Colesterol solamente
metacarpo falángicas y muñeca. La B) LDL – Colesterol solamente
radiografía muestra osteopenia periarticular C) Colesterol total solamente
y erosiones de las articulaciones D) Relación de Colesterol con HDL Colesterol
interfalángicas proximales y metacarpo E) Nivel de triglicéridos solamente
falángicas. El paciente inició tomando 33) Un hombre pálido de 50 años acude a la
indometacina, que provee buen alivio, pero oficina y dice: “por el último año me he
los síntomas recurren si se salta una dosis. sentido muy débil, cansado muy temprano y
¿Cuál de las siguientes es el próximo paso siento que estoy perdiendo mi sentido del
más apropiado en el manejo de este humor. Se me hace difícil recordar las cosas
paciente? ahora”. Cuando se le pide sobre sus
A) Azatioprina antecedentes sociales comenta “soy
B) Celecoxib desempleado y vivo de los beneficios del
C) Etanercept estado. No bebo ni fumo ya, pero se me
D) Glucocorticoides hicieron cargos por conducir bajo efectos del
E) Metotrexate alcohol en el pasado en tres ocasiones”. Sus
padres fallecieron por vejez. El comparte el
hogar con 4 amigos. Sus signos vitales están B) Cirugía de urgencia
normales. El hemograma revela: Glóbulos C) Radiografía abdominal
blancos: 5,500/mm3 , Hemoglobina: D) Analgesia con opioides
7mg/dl, Hematocrito: 22%, plaquetas: E) Sigmoidoscopia con biopsia
196,000/mm3. Eritrocitos: 1.7
millones/mm3, VCM: 119 fl. HCM: 36. MCHC: 35) Un hombre de 32 años es admitido al
28%. Reticulocitos: 0.4% Frotis de sangre hospital por ser encontrado al borde de un
periférico muestra anisocitosis, poikilocitosis puente. Impresiona severamente deprimido,
y punteado basófilo. ¿Cuál es el próximo anunciando que su vida está acabada. El
paso a seguir en el manejo de este paciente? está arruinado económicamente y su esposa
A) Estudios de hierro recientemente lo abandonó, tomó a sus hijos
B) Prueba de fragilidad osmótica y los llevó con ella. El ha estado llorando,
C) Niveles de folato y vitamina B12 incapaz de dormir, sin apetito y
D) Prueba de agua dulce continuamente expresa sus sentimientos de
E) Biopsia de médula ósea con tinción de azul desesperanza y pensamientos suicidas. Se le
de Prusia inició sertralina y clonazepan para sus
F) Nivel de plomo en sangre síntomas. Más tarde en ese día, el paciente
se vuelve molesto e inicia golpeándose la
34) Un hombre de 26 años acude con historia cabeza repetidamente contra el borde del
de 4 semanas de dolor abdominal en los lavamanos. Se le administran inyecciones
cuadrantes inferiores tipo intermitente y con intramusculares de lorazepan y haloperidol
calambres acompañado de urgencia rectal, para reducir su agitación y fue físicamente
diarrea sanguinolenta, náuseas y disminución restringido por un breve periodo de tiempo.
del apetito. Sus síntomas se han vuelto más El continua agitado los días siguientes,
severos en los últimos 2 días. No tiene requiriendo lorazepan y haloperidol de
antecedentes médicos y niega uso de emergencia unas tres veces más y restricción
antibióticos o viajes recientes. Su física unas 2 veces más. En el tercer día, el
temperatura es de 38.5°C, presión sanguínea paciente notablemente más calmo, es
de 90/50mmHg, pulso de 130/min y incapaz de salir de cama. Impresiona
respiración de 16/min. El examen abdominal confundido y no responde a las preguntas en
revela distención, dolor difuso a la palpación forma apropiada. Está diaforética y su
y disminución de los ruidos intestinales. Al temperatura es 39.5°C. Su presión arterial es
examen rectal muestra dolor rectal y una 167/97mmHg y su pulso en 112/min. Como
mezcla de sangre y como en la ampolla. Los el personal trató de sacarlo de la cama,
laboratorios se muestran: notaron que sus brazos están rígidos y
Hemoglobina: 10.2gr/dl difíciles de doblar. ¿Cuál de los siguientes es
Leucos: 31,600/mm3 el más probable causal de estos síntomas?
Plaquetas: 459,000/mm3 A) Episodios de golpearse la cabeza
VES: 54/h. Se le inicia fluidos intravenosos. B) periodos de restricción física
¿Cuál de los siguientes es el paso más C) Haloperidol
apropiado en el manejo de este caso? D) Sertralina
A) Enema de Bario
E) empeoramiento de los síntomas los cuadrantes inferiores con defensa
psiquiátricos. moderada. No hay matidez cambiante y los
sonidos intestinales están disminuidos. Los
36) Una mujer de 51-años de edad, G3P3 se hallazgos de laboratorio son:
presenta a su oficina con una historia de 6 Hemoglobina: 13.1 g/dl
meses de amenorrea. Ella se queja de Hematocrito 43%
sofocos que la despiertan en la noche; al día Leucocitos 10,900/mm3
siguiente amanece sintiéndose agotado e Cuál de los siguientes es el paso más
irritable. Ella dice que ha intentado con apropiado a seguir en el manejo?
suplementos de hierbas para los sofocos, A) Tomografía abdominal
pero nada hace efecto. Está interesada en B) Lavado peritoneal diagnóstico
iniciiar terapia de reemplazo hormonal, pero C) Ultrasonido pélvico
es reacia a hacerlo debido a sus posibles D) Prueba de embarazo
riesgos y efectos secundarios. La paciente es E) Radiografía de abdomen de pie
muy saludable, niega cualquier problema
médico y no está tomando ninguna 38) Un hombre de 59 años es llevado a
medicamentos, excepto los suplementos de urgencias debido a confusión progresiva en
calcio. Ella tiene un historial familiar de los últimos 2 días. Ha tenido varios episodios
osteoporosis. Su altura es de 165 cm Y su de vómitos sin hematemesis. Según la
peso es de 115 lbs. Al aconsejar al paciente esposa, se ha quedado de cefaleas severas y
sobre los riesgos y beneficios de la terapia de poco sueño recientemente. Seis meses
reemplazo hormonal, le dices que cuál de los antes, fue diagnosticado con cáncer
siguientes es un riesgo documentado de la prostático localizado que fue tratado con
TRH (estrógeno y progesterona)? cirugía. Sus antecedentes médicos incluyen:
A) Aumento de cáncer de colon hipertensión, gota, cefalea tensional y asma
B) Aumento de cáncer de útero leve intermitente. No fuma pero su esposa
C) Aumento de eventos tromboembólicos ha notado que ha estado bebiendo 2 – 3
D) Aumento de la enfermedad de Alzheimer “shots” de whiskey cada noche. No tiene
E) Aumento en el melanoma maligno alergias conocidas. Al examen físico hay
hepatomegalia. No hay ascitis ni edema
37) Una mujer de 38 años es llevada a periférico. Los laboratorios muestran:
urgencias por su novio para que sea evaluada Albúmina: 3.2 mg/dl
por dolor abdominal severo. El dolor inició Bilirrubina total: 6.7mg/dl
de forma difuso en la región periumbilical Aspartato aminotransferasa (AST, SGOT):
varias horas antes pero ahora se ha 6245 UI/L
localizado sobretodo en el abdomen bajo Ella Alanino aminotransferasa (ALT, SGPT): 7116
se queja de náusea persistente pero no hay UI/L
vómitos. Su último periodo menstrual fue Tiempo de protrombina: 28 seg
hace 25 días. Su temperatura es 36.8°C, Tiempo de tromboplastina: 43 seg
presión arterial de 130/80 mmHg, pulso en Cuál de las siguientes es el diagnóstico más
110/min, respiración de 25/min. Abdomen probable de este paciente?
es doloroso a la palpación, especialmente en A) Hepatitis alcohólica aguda
B) Cirrosis hepática suplementario. ¿Cuál de las siguientes
C) Daño hepático inducido por hubiese sido más efectiva estrategia en
medicamentos prevenir su condición actual?
D) Malignidad metastásica a hígado A) Ejercicios respiratorios
E) Esteatosis hepática no alcohólica B) Antibióticos de amplio espectro
C) Heparina de bajo peso molecular
39) Un hombre de 46 años es hallado D) Cesación de tabaquismo 1 semana previo
taquineico en su segundo día de reparación a la cirugía
de hernia ventral en el abdomen superior. E) Glucocorticoides sistémicos y albuterol
No tiene dolor torácico. La cirugía no tuvo
complicaciones, pero en la tarde el paciente 41) Un paciente varón de 18 años acude a
se quejó de dolor abdominal y solicitó consulta con por dolor con pesadez y
calmantes. El tiene una tos crónica con algo sensación de llenura en el escroto. Al
de esputo matutino. Sus otros problemas examen muestra un edema suave del teste
médicos incluyen asma leve persistente, izquierdo. La prueba de transluminación es
hipertensión e hiperlipidemia. El paciente es negativa. El edema escrotal aumenta cuando
un fumador activo de 25/paquetes /año. Su el paciente realiza la maniobra de valsava.
temperatura es de 36.7°C, presión arterial de Por lo demás está normal. ¿Cuál de los
123/79mmHg, pulso de 90/min, respiración siguientes es la causa más probable de esta
de 28/min. Su índice de masa corporal es de condición?
32Kg/m2. El examen muestra disminución de A) Hipoalbuminuria
los ruidos pulmonares a la base derecha sin B) Dilatación del plexo pampiniforme
sibilancias ni prolongación de la espiración. C) Líquido en la túnica vaginalis
Los sonidos cardiacos están normales. Hay D) Neoplasia testicular
algo de distensión abdominal y leve dolor E) Dilataciones quísticas de los ductos
difuso sin rebote. La radiografía muestra una eferentes.
opacidad densa en las bases pulmonares.
La gasometría a aire ambiente reflejaría cuál 42) Un hombre de 55 años de edad es traído
de las siguientes? a emergencias por repentina instauración de
palpitaciones y sensación opresiva torácica.
Sus antecedentes médicos incluyen
hipertensión, gota y diabetes tipo 2. El
monitor cardiaco muestra fibrilación atrial
con una frecuencia de 120-140/min. Su
presión inicial es de 112/70mmHg y la
saturación de oxígeno 92% aire ambiente.
Mientras la enfermera intenta lograr un
acceso venoso, el paciente cae en un estado
40) Un electrocardiograma realizado al en el que no responde a las preguntas de la
paciente anterior muestra ritmo sinusal sin enfermera. No hay pulso palpable sobre las
segmentos agudos ST ni cambios en las carótidas o las arterias femorales e inicia una
ondas T. Al paciente se le brinda oxígeno respiración agónica. El monitor cardiaco aún
muestra la fibrilación atrial en el mismo dolor epigástrico. Hay una masa palpable
rango de frecuencia. ¿Cuál de los siguientes debajo del margen costal derecho. Signo de
es el mejor próximo paso a seguir en el Murphy negativo. El ultrasonido muestra
manejo del paciente? una vesícula biliar agrandada y dilación de
A) Análisis de gases arteriales los ductos hepáticos. Los laboratorios
B) Compresiones torácicas muestran:
C) Desfibrilación Glóbulos blancos: 11,000/mm3,
D) Lidocaína IV Bilirrubina Total: 2.4mg/dl,
E) Cardioversión sincronizada Bilirrubina directa: 1.6mg/dl,
Fosfatasa alcalina: 310 U/L,
43) Un hombre de 42 años acude al médico Amilasa: 140 UI/L,
por evaluación de rash cutáneo y caída del Aspartato aminotransferasa (SGOT): 87 UI/L,
cabello. Tiene antecedente de Enfermedad Aspartato aminotransferasa (SGPT): 56 UI/L
de Crohn de larga evolución y tuvo una ¿Cuál de los siguientes es el paso más
resección extensa del intestino delgado con apropiado a seguir en este caso?
el resultante síndrome de intestino corto. A) Radiografía abdominal simple
Actualmente recibe nutrición parenteral B) Colangiopancreatografía retrógrada
total. Cuando trata de comer, se queja de endoscópica
que la comida no sabe a comida. Sus signos C) Colangiografía percutánea transhepática
vitales están estables. Al examen muestra D) Tomografía abdominal
alopecia y lesiones bulosas y pustulares E) Centelleo HIDA para obstrucción de ducto
alrededor de las áreas perioral y periorbital. biliar.
¿Cuál de los siguientes es la causa más
probable de los síntomas actuales del 45) Una mujer de 27 años de edad, acaba de
paciente? tener un embarazo ectópico. ¿Cuál de los
(A) Enfermedad celiaca siguientes factores tiene mayor probabilidad
(B) Deficiencia de zinc para predisponer a un embarazo ectópico?
(C) Deficiencia de vitamina A A) Cirugía tubárica previa
(D) Deficiencia de vitamina B12 B) Enfermedad inflamatoria pélvica (EIP)
(E) Deficiencia de selenio C) Uso de un dispositivo anticonceptivo
uterino (DIU)
44) Una mujer de 58 años acude a urgencias D) Inducción de la ovulación
por dolor intermitente con pesadez sub E) Exposición en el útero a dietilestilbestrol
costal derecha, intolerancia a las comidas y (DES)
pérdida de 13.2 lbs en 2 meses. Su dolor ha
ido empeorando sobretodo en los últimos 2 46) A los 23 años de edad, una paciente
días. Inicialmente era localizado en el G1P0 se presenta sala de labor y parto con
cuadrante superior derecho, pero ha 39 semanas quejándose de contracciones
cambiado hacia su espalda. Sus uterinas irregulares durante las últimas
antecedentes médicos incluyen apendicitis horas, algunos de los cuales son dolorosas. El
(complicada con peroración y peritonitis malestar se encuentra principalmente en la
hace 20 años). El examen abdominal hay parte baja del abdomen. Ella reporta buenos
movimientos fetales y niega cualquier D) Eritema multiforme
sangrado vaginal o salida de líquido. Las E) Dermatitis herpetiforme
enfermeras colocan a la paciente en un
monitor fetal externo. El monitor indica que 48) Un nuevo esquema de quimioterapia
se está contrayendo cada 2 a 10 minutos, y multidroga prolonga significativamente la
las enfermeras reportan que las supervivencia de pacientes con cáncer
contracciones son leves a la palpación. El pulmonar. Si este nuevo régimen es
examen del cuello uterino es 50 / 1 / -1, ampliamente implementado, cuál cambio en
vértice. Este examen no ha cambiado desde la prevalencia y la incidencia de cáncer de
su última consulta hace 1 semana. El pulmón usted esperaría?
monitoreo fetal es un trazo reactivo sin A) Incidencia y prevalencia disminuiría
deceleraciones. El paciente está cansada de B) Incidencia aumentaría y prevalencia no
estar embarazada y quiere que el bebe nazca aumentaría
lo más pronto posible. C) Incidencia no cambiaría y prevalencia
¿Cuál es el diagnóstico más probable de este aumentaría
paciente? D) Incidencia y prevalencia no cambiarían
A) Labor activa E) Incidencia disminuiría y prevalencia
B) Labor latente aumentaría
C) Falsa Labor
D) Etapa 1 de labor 49) Una mujer de 44 años acude al médico
E) Etapa 2 de labor por 1 mes de cefalea severa que no se alivia
con medicinas analgésicas de libre compra.
47) Un hombre de 55 años acude a la clínica La cefalea se localiza en el centro de su
por instauración aguda de ampollas sobre cabeza. Ha sido constante y es peor en las
todo su cuerpo. Se queja de dolor en las mañanas. Niega fiebre, debilidad,
áreas involucradas. Las notó por primera vez sensaciones anormales, convulsiones,
hace unos días atrás en su boca. Su pulso es pérdida de conciencia. Tiene antecedentes
82/min, presión arterial 140/90 mmHg, de 15 años de cefaleas tensionales que se
respiración en 14/min. Temperatura de alivia con acetaminofén, tiene antecedente
36.8°C. A la evaluación hay bulas flácidas y de cáncer de mama. Tuvo mastectomía
duras sobre piel de apariencia normal, con radical modificada y varios cursos de
sitios grandes de erosión donde las bulas se quimioterapia hace 2 años. ¿Cuál de los
han roto. La mucosa oral muestra erosiones siguientes es el próximo paso en el manejo
y ulceraciones. El frote suave sobre la piel no de la paciente?
involucrada produce una fácil separación de A) Administración de dexametazona
la epidermis. La microscopia por B) Tomografía cerebral con contraste
inmunofluorescencia muestra depósitos de C) Tomografía cerebral sin contraste
IgG intercelular en la epidermis. ¿Cuál es el D) Radiación externa de tallo
diagnóstico más probable? E) Consulta a neurocirugía
A) Pénfigo buloso F) Propranolol
B) Impétigo buloso G) Sumatriptan
C) Pénfigo vulgaris H) Radiografía de tórax AP y lateral.
50) Un hombre inmigrante de Colombia de 51) Una mujer de 28 años acude al médico
74 años acude a su consulta y se queja de por consulta de rutina. Tiene antecedente
“tener toda clase de problemas de la vejez”. de asma bronquial de 2 años para lo cual usa
Tiene dolor en todo su cuerpo. Con un inhalador de albuterol. Refiere que tiene
frecuencia tiene cefaleas y se siente síntomas de asma en un promedio de 2
mareado. Tiene problemas visuales, y se le veces / semana la cual se alivian con el uso
hace difícil caminar. En las últimas semanas del albuterol. No tiene síntomas ni
ha estado sintiendo sus pies débiles y despertares nocturnos por el asma en los
adormecidos. Vive con su hijo, y no se siente últimos meses. El no fuma, no bebe alcohol
feliz en la forma en que su hijo lo trata, sin ni usa drogas ilícitas. Tiene antecedente de
embargo niega recibir algún tipo de abuso asma en su abuelo. A la evaluación física no
físico. No tiene antecedentes médicos. Su hay hallazgos patológicos relevantes y los
madre tiene “algún tipo de problema o parámetros clínicos se encuentran dentro de
enfermedad de la sangre”. Sus signos vitales los rangos normales. ¿Cuál de los siguientes
son estables. El examen físico revela es el mejor próximo paso a seguir en el
múltiples hematomas en su cuerpo, déficit manejo de esta paciente?
sensorial en sus pies. Otros hallazgos A) Adicionar un inhalador beta 2 agonista de
significados son linfadenopatías y acción larga
hepatoesplenomegalia. El fondo de ojo B) Adicionar un inhalador de corticosteroides
muestra venas retinianas, tortuosas, C) Adicionar teofilina oral
segmentadas y dilatadas. Los estudios de D) Adicionar prednisona oral
laboratorio obtenido son: E) Continuar con el tratamiento actual sin
Glóbulos blancos: 10,200 /mm3 modificaciones
Hemoglobina: 9.6g/dl
Hematocrito: 29% 52) Un varón de 6 años se presenta al
Plaquetas: 94,000/mm3 hospital después de 2 años de hematuria
Sodio: 141 mEq/L macroscópica recurrente NO hay disuria o
Potasio: 3.6 mEq/L piuria. El examen de orina no muestra
Nitrógeno de urea: 18 mg/dl glóbulos blancos y el cultivo es estéril. Los
Creatinina: 0.8 mg/dl niveles de C3 son normales. El diagnóstico
Glucosa: 115 mg/dl más probable es:
La electroforesis de proteínas muestra un A) Tumor Wilms
pico de IgM. ¿Cuál de los siguientes es el B) Nefropatía por IgA
diagnóstico más probable de este paciente? C) Glomerulonefritis post estreptococo
A) Mieloma múltiple D) Infección urinaria por E. Coli
B) Macroglobulinemia de Waldestrom E) Infección Urinaria por Klebsiella spp
C) Gamapatía monoclonal de significancia
indeterminada 53) Sobre la Acalasia (cardias) es cierto:
D)) Abuso de adulto mayor A) Disfagia es un síntoma que rara vez está
E) Enfermedad de cadenas pesadas presente
B) La causa es la ausencia del plexo Auerbach
C) Esofagectomía es el tratamiento
D) Medicamentos para mejorar la motilidad 56) Un hombre de 35 años, trabajador en
son coadyuvantes del tratamiento una oficina postal, se ha estado perdiendo en
E) Trago de bario muestra defecto de llenado lugares familiares, que eran conocidos por
irregular en el esófago inferior años. Ha estado de mal humor, irritable e
irracional. Los miembros de su familia han
54) Un hombre de 75 años requiere aumento notado que “toma malas decisiones”. Su
de los parámetros ventilatorios después de hermana de 42 años está en un albergue por
varios días de requerir ventilación mecánica una enfermedad neurodegenerativa crónica.
por falla respiratoria. Sus rayos X muestran Su padre murió 10 años antes por la misma
infiltrados bilaterales. Basados en sus condición. Cuál de los siguientes sería la
parámetros ventilatorios la relación de prueba más utíl para el diagnóstico.
Presión parcial de oxígeno y la fracción A) Electroencefalograma
inspirada de oxígeno es 190. Cuál es la B) Electrocardiograma
etiología subyacente más probable del C) Prueba de función hepáticas
síndrome de distres agudo respiratorio? D) Prueba de supresión con dexametasona
A) Broncoaspiración E) Resonancia magnética
B) Sobredosis droga
C) Sepsis 57) Mujer de 50 años, previamente sana se
D) Transfusión sanguínea masiva prsenta con dolor muscular progresivo y
E) Ahogamiento debilidad por 3 semanas. Los síntomas son
localizados principalmente en músculos
55) Una niña de raza negra de 5 días de proximales afectando el deltoides y músculos
nacida es llevada al pediatra porque sus ojos pélvicos. Al examen físico hay dolor a la
están amarillos. Es alimentada por fórmula palpación de los músculos afectados y
exclusivamente, enriquecida con hierro. objetiva pérdida de fuerza muscular. Los
Moja 6 pañales diarios y evacua 2 veces al laboratorios revelan creatinin kinasa en 2000
día. Embarazo sin complicaciones. Nacida por UI/L. Electromiografía muestra cambios
parto vaginal eutócico, sin complicaciones. miopáticos. La biopsia deltoides revela
Apgar 9/10. Al examen físico con atrofia y necrosis dispersa de miofibrillas con
temperatura de 37°C , su perímetro cefálico infiltración inflamatorio endomisial
está en el percentil 50, su peso es 3420g (3 g principalmente compuesto de linfocitos.
por debajo del peso al nacer). Tiene escleras ¿cuál de las siguientes es el diagnóstico más
ictéricas, No hay hepato ni espleno megalias. propable?
Su bilirrubina total es de 9mg/dl, Su A) Denervación atrófica
bilirrubina conjugada es de 0.2mg/dl, Su B) Dermatomiositis
hemoglobina es de 15g/dl. Cuál de los C) Miositis de cuerpos de inclusión
siguientes es el diagnóstico más probable: D) Polimiositis
A) Deficiencia de alfa 1 antitripsina E) Lupus eritematoso sistémico
B) Atresia biliar
C) Síndrome de Dubin-Johnson 58) Un hombre de 67 años es evaluado por
D) Ictericia fisiológica hipertensión. Se queja de ocasionales
E) Síndrome de Rotor dolores de cabeza matutinos. Entre sus
antecedentes están Diabetes mellitus tipo 2,
enfermedad de arterias coronarias, y un
evento cerebrovascular con secuela de
debilidad / paresia del lado izquierdo. Tuvo
una cirugía de bypass coronario hace años y
una endarterectomía carotídea hace 5 años.
Actualmente sus medicamentos incluyen
lisinopril, hidroclorotiazida, amlodipina,
metoprolol, aspirina, metformina y gliburida.
Su presión arterial es 190/120 mmHg en el ¿Cuál de la siguiente es el mejor tratamiento
brazo izquierdo y 170/110mmHg en el brazo para este paciente?
derecho,. Su pulso es de 65/min. Al examen A) Cobalamina
físico hay un soplo sistólico-diastólico B) Asegurar que no requiere terapia
periumbilical, lo cal es mejor explicado por específica
cuál de las siguientes? C) Hierro
A) Aneurisma de la aorta abdominal D) ácido fólico
B) Coartación aórtica E) Hidroxiurea
C) Disección aórtica
D) Fístula aortoentérica 60) Un niño de 2 años de edad es llevado al
E) Estenosis de arteria renal médico por sus padres por rash pruriginoso
que ha sido un problema persistente desde
59) Un hombre italiano – americano de 32 hace 6 meses. El rash es seco, eritematoso y
años acude al médico por evaluación de escamoso e involucra principalmente la cara,
rutina. Es ejecutivo de empresa y ha estado la superficie dorsal de las manos y pies. Las
bajo mucho estrés recientemente. Bebe lesiones alternan periodos de remisión y
alcohol ocasionalmente, fuma 10 paquetes / exacerbación. El tratamiento con esteroides
año. Los laboratorios de seguimiento son: locales produce alivio temporal. Al examen
Hemoglobina 10.8 g/dl se revela piel seca con marcas de la piel muy
Eritrocitos 5.7/uL pronunciadas en las superficies palmares y
VCM: 61 fl plantares. La historia familiar es significativa
Leucocitos: 5,500/uL por rinitis alérgica y asma en varios
Plaquetas: 170,000 / uL parientes. El desarrollo del niño es por lo
Sangre oculta en heces seriada es negativa. demás normal. Cuál es el diagnóstico más
El frotis de sangre periférica es el mostrado probable?
en la imagen A) Dermatitis alérgica de contacto
B) Dermatitis atópica
C) Histiocitosis de Células de Langerhans
D) Eczema numular
E) Escabiasis

61) Estudiante, de primer año de


universidad, de 19 años de edad,
homosexual, acude a consulta por diarrea de persistente, pérdida de peso de varios meses
2 días de duración. Hace 9 días regresó de de evolución. Niega antecedentes de
un viaje en México. Su diarrea se acompaña tabaquismo pero ha trabajo en áreas
de marcada distensión abdominal, subterráneas en construcción por más de 20
flatulencia, náuseas y malestar general. Al años. La espirometría realizada muestra una
examen físico impresiona bien hidratado, relación FEV1/FVC de 0.7, un FEV1 del 60%
con temperatura de 36.9°C, presión arterial del esperado. El FEV1 mejora a 70% con
de 130/86mmHg, pulso de 89/min y broncodilatador. Cuál de los siguientes es el
respiración de 18/min. Al examen abdominal diagnóstico más probable?
los ruidos intestinales están difusamente A) Asma
hiperactivos sin zonas dolorosas o masas. La B) Aspiración crónica
muestra de heces es negativa por glóbulos C) Enfermedad pulmonar obstructiva crónica
rojos y blancos. ¿Cuál de los siguientes es el D) Histoplasmosis
manejo más apropiado para el caso? E) Tuberculosis
A) Tratamiento con líquidos intravenosos
B) Tratamiento con ciprofloxacina 64) Una mujer de 66 años acude para
C) Tratamiento con mebendazole remoción de lipoma en el codo. Ella quiere
D) Tratamiento con Metronidazole removerlo dado que cuando utiliza mangas
E) Tratamiento con Trimetropin - cortas se ve “feo”. Su únicas quejas son
Sulfametoxazole malestar general y fatiga en los últimos 8
meses lo cual atribuye a que ha “estado sola
62) Una mujer de 65 años con una historia todo el tiempo”. Sus signos vitales están
prolongada de abuso de alcohol, saluda dentro de límites normales. Al examen físico
emocionada al médico del hogar, quien ha hay leve palidez y linfoadenopatías tanto
visto muchas veces, llamándole “su querido cervical como supraclavicular. Su
amigo Jack”. El doctor le explica a la hemograma preoperatorio muestra:
paciente quien es y le dice su nombre. Dos Hemoglobina de 10.0 g/dl
minutos después, cuando le pregunta al Hematócrito en 32%
paciente si ella conoce quién es el, ella con Plaquetas en 101,000/mm3
una gran sonrisa dice: “ si claro, eres mi Glóbulos blancos en 41,600/mm3
primo Anthony de Nueva Jersey”. Cuál Neutrófilos segmentados 18%
deficiencia vitamínica puede causar esta Linfocitos 77%
particular forma de desoden amnésico Bandas 4%
inducido por alcohol? Monocitos 1%
A) Ácido pantoténico El patólogo reporta la presencia de
B) Folato leucocitos que han sido parcialmente
C) Tiamina fragmentados durante la preparación del
D) Riboflavina frotis y tinción, debido a su gran fragilidad.
E) Niacina La biopsia de nódulo confirma el diagnóstico.
¿Cuál es la oración correcta sobre el paciente
63) Un hombre de 55 años se presenta a la descrito?
clínica por dificultad respiratoria, tos A) La sobrevivencia media esmenos de un
año que revele:
B) Esta es una forma de leucemia de células A) Fusión de los procesos pedios de las
plasmáticas células epiteliales glomerulares
C) La presencia de trombocitopenia es un B) Rarefacción de la membrana basal
factor de mal (pobre) pronóstico glomerular
D) Es clásicamente una enfermedad de C) Depósito de material denso en la
células T membrana basal
E) La causa más común de muerte es la falla D) Adelgazamiento de la membrana basal
renal E) Depósito de material ligero en la
membrana basal
65) Un niño de 3 años de edad, es llevado a
su pediatra por fiebres crónicas de bajo 67) Un hombre de 65 años acude al médico
grado, disminución del apetito y tos no debido a fiebre, escalofríos y tos productiva.
productiva. Ha sido previamente sano, sin Sus síntomas iniciaron hace 4 días atrás.
episodios similares del pasado. El pediatra se También refiere dolor torácico que empeora
entera de que el paciente ha estado al inspirar. Es fumador de 1 paquete de
visitando a sus abuelos en un centro de cigarrillos / día por los últimos 45 años. Bebe
retiro los fines de semana. El médico quiera de 4 a 5 onzas de alcohol de forma diaria. Su
descartar todas las posibilidades y ordena radiografía de tórax muestra un infiltrado en
una radiografía del tórax, la que parece ser el lóbulo pulmonar superior derecho. La
normal. Una prueba de derivado de proteína evaluación del esputo revela un bacilo gran
purificada (PPD) se lleva a cabo y 48 horas negativo encapsulado. El cultivo lleva un
más tarde demuestra 12-mm de induración. crecimiento de colonias mucoides. ¿Cuál de
El diagnóstico de la tuberculosis pulmonar se los siguientes es el microorganismo causal
hizo, y el niño inicia tratamiento con más probable en este paciente?
isoniazida, rifampicina y pirazinamida. ¿Cuál A) Escherichia coli
de las siguientes es una reacción adversa B) Streptococcus pneumoniae
conocida de rifampicina que debe vigilarse C) Klebsiella pneumoniae
durante el tratamiento de este niño? D) Legionella spp.
(A) Angioedema E) Mycoplasma pneumoniae
(B) Hepatotoxicidad
(C) Hiperuricemia 68) Un niño de 13 años de edad con
(D) Neuritis periférica enfermedad de células falciformes es llevado
(E) síndrome de Stevens-Johnson a la sala de urgencias para la evaluación por
dolor en las piernas. Dice que desarrolló un
66) Un niño de 7 años, se presenta con dolor significativo en la pierna hace
edema generalizado. El examen de orina aproximadamente 2 días que no le permitió
presenta marcada albuminuria. La su participación en la clase de gimnasia e
bioquímica sérica muestra hipoalbuminemia, incluso creó marcado dolor al caminar. Ha
hiperlipidemia. Se realiza biopsia renal que a tomado solo paracetamol para el dolor con
la microscopia de luz se observa normal, Al una leve mejoría. Sin embargo, en las últimas
examen microscopio electrónica es probable 24 horas han notado que el área del dolor
empieza a mostrar enrojecimiento de la piel ALT: 102 UI/L
con un poco de hinchazón. Su ritmo cardíaco Fosfatasa Alcalina: 180 UI/L
es 92 / min, presión arterial es 108/68 mm Bilirrubina total 1.9 mg/dl
Hg, frecuencia respiratoria es de 19 / min, y Al examen físico: hepatomegalia, ligero tinte
la temperatura es de 38,5 ° C . El examen ictérico en piel. En la revisión sistemática
físico es normal, excepto por dolor en la tibia refiere: pérdida de peso, debilidad, artralgia
derecha con edema leve y eritema de la piel en manos, disminución de la libido. ¿Cuál
que lo recubre. Él es enviado para una estudio debe realizarse primero para la
evaluación radiológica y se encontró que no sospecha diagnóstica de este caso?
tiene lesiones líticas, pero que demuestra A) Hemograma completo
edema del músculo profundo. ¿Qué B) Tomografía de abdomen
organismo es más probable responsables del C) Colangiopancreatografía retrógrada
estado de este niño? endoscópica
(A) Escherichia coli D) Nivel de saturación de transferrina
(B) Haemophilus influenzae E) Biopsia hepática
(C) Mycoplasma
(D) Salmonella 71) Un hombre de 25 años acude al médico
(E) Staphylococcus aureus por instauración rápida de dolor y edema de
su rodilla izquierda, que inició hace apenas
69) Un pediatra, varón de 70 años, desarrolla 24 horas. Su temperatura es de 38.5°C,
de forma repentina cefalea, hablar enredado presión arterial de 125/70mmHg, pulso de
y confusión. Es llevado a urgencias donde la 95/min y respiración de 20/min. En su
escala Glasgow de coma es del 8. La historia personal refiere uso de drogas IV.
tomografía cerebral de urgencia revela un Niega historia de enfermedades de
sangrado lobar bien demarcado que transmisión sexual. La rodilla izquierda está
involucra el lóbulo frontal izquierdo, dolorosa, edematizada y caliente al tacto. La
centrado en la corteza y se extiende a la auscultación torácica es normal. ¿Cuál de las
materia blanca subyacente. El paciente no siguientes es el próximo paso en el manejo?
tiene antecedente de hipertensión o factor A) Laboratorios en sangre, incluyendo
de riesgo vascular. ¿Cuál de los siguientes es hemograma completo
la causa subyacente más probable del B) Prueba de HIV antes de iniciar cualquier
sangrado del paciente? tratamiento
A) Ruptura de un aneurisma en cereza C) Rayos X simple de rodilla
B) desgarro de venas del puente D) Antiinflamatorios no esteroidales y
C) Angiopatía amiloide cerebral tratamiento antibiótico empírico
D) Laceración de la arteria meningea media E) Artrocentesis diagnóstica
E) Hipertensión no diagnosticada
72) Después de un primer embarazo que
70) Un hombre de 63 años diabético, terminó en aborto espontáneo en el primer
presenta elevación de las pruebas de función trimestre, su paciente está preocupada sobre
hepática: la posibilidad de la recurrencia. La respuesta
AST: 85 UI/L apropiada sobre la posibilidad de recurrencia
debe ser: que la pérdida de apetito es un efecto
A) Depende de la genética de los abortos adverso común del metilfenidato. Otro
previos efecto adverso serio de la medicación sería:
B) No es diferente o mayor que el aborto A) Terrores nocturnos
previo B) Movimientos coreiformes
C) Se aumenta el riesgo un 50% C) Tics
D) El riesgo aumenta más del 50% D) Arritmias cardiacas
E) Depende del sexo del aborto previo E) Leucopenia

73) Una mujer de 55 años de edad se 75) El valor de corte de los niveles de
presenta con su ginecólogo por molestias normalidad del antígeno prostático
vaginales. Comenzó hace aproximadamente específico (PSA) es de 4,0 ng / dl. Si este
6 meses y ha fluctuado en intensidad, más valor se disminuyera a 3,7 ng / dl, la
recientemente, causando picazón y un ligero sensibilidad y especificidad de la prueba para
dolor cuando tiene relaciones sexuales con la detección de cáncer de próstata va a
su marido. Ella fuera de eso ha estado bien. cambiar. ¿Cómo la reducción del valor
Es postmenopáusica desde hace 3 años. Su normal de PSA en el cribado del cáncer de
hermana mayor le recomienda una crema de próstata alteraría el número de falsos
estrógeno, que ha utilizado constantemente positivos y falsos negativos?
durante más de 3 meses sin ningún cambio A) Disminuir tanto falsos positivos y falsos
en los síntomas. negativos
En el examen introito vaginal, la mucosa y los (B) Disminuir el número de falsos negativos y
labios menores son de color blanco-rosado, aumentar el número de falsos positivos
con arrugas / pliegues anormales y una (C) Aumentar tanto falsos positivos y falsos
pequeña fisura en los labios menor derecho. negativos
¿Cuál de los siguientes es el diagnóstico más (D) Aumentar el número de falsos negativos
probable? y disminuir el número de falsos positivos
A) Candidiasis (E) No sería cambiar alguno de los
B) Deficiencia estrogénica parámetros
C) Liquen escleroso
D) Abuso sexual 76) Un paciente de 23 años de edad con un
E) Vitiligo historial médico de diabetes tipo 1 y
tratamiento ambulatorio reciente para la
74) Un varón de 6 años ha sido recién enfermedad inflamatoria pélvica con
diagnósticado con trastorno de antibióticos orales regresa al servicio de
hiperactividad con déficit atencional, se le urgencias por fiebre y dolor abdominal bajo
inicia metilfenidato 5mg tres veces / día. Los durante 1 día. Su temperatura es de 39,6°C,
padres reportan que el niño ha sido capaz de pulso es de 125/min, presión arterial es
enfocarse mejor, es menos hiperactivo y 88/48 mmHg, y la frecuencia respiratoria es
parece ser mas tolerante a la frustración. de 18/min. Ella confirma que tomó toda
Están preocupados sin embargo, por que terapia con antibióticos orales y
nunca tiene hambre. El psiquiatra le explica regularmente ha estado usando su insulina.
En el examen de la piel está caliente al tacto tiene una lesión en el lóbulo del oído
y su abdomen es rígido y con dolor difuso a izquierdo. Ella lo noto por primera vez hace
la palpación superficial. Las pruebas de 6 meses y refiere que está al doble de su
laboratorio muestran: tamaño inicial. Es indoloro. Ella trabaja
Hemoglobina 13.4 g/dL como jardinera y está expuesta a la luz del
Glóbulos blancos; 24,000/mm3 sol, niega utilizar sombrero. La lesión es fija,
Plaquetas en 247,000/mm3 con una úlcera central con bordes
Glucosa plasmática 224 mg/dL relucientes y amontonados. ¿Cuál de los
Urinálisis es negativo por β-Gonadotropina siguientes es el diagnóstico más probable?
coriónica humana. ¿Cuál de los siguientes es A) Carcinoma de células basales
la causa más probable de sus síntomas B) Eritema nodoso
subyacentes? C) Leucoplaquia
A) Colecistitis Aguda D) Melanoma
B) Cetoacidosis diabética E) Carcinoma de células escamosas
C) Cervicitis gonorreica después de
reinfectarse por compañero sexual no 79) Un hombre de 37 años está siendo
tratado evaluado luego de un accidente vehicular. Se
D) Ruptura de embarazo ectópico queja de dolor torácico de lado derecho. A la
E) Ruptura de absceso tubo-ovárico evaluación hay un leve escoriación en la
pared torácica derecha, fuera de eso no hay
77) Un niño de 11 años de edad que ha otro hallazgo. La radiografía torácica no
estado aislado, desafiante, abatido, y muy muestra fracturas costales pero se aprecia
irritable durante varias semanas se ahorca en una lesión solitaria redondeada en el lóbulo
el garaje después de que sus padres superior derecho del pulmón. En la
amenazan con regalar a su amado perro si él entrevista el paciente niega pérdida de peso
"no da la talla." Elegir la afirmación correcta: reciente o cambios en el apetito. No ha
A) La depresión es poco frecuente en niños fumado nunca. ¿Cuál de los siguientes es el
B) La presentación de la depresión en los más apropiado paso a seguir en el manejo de
niños es a menudo similar a la presentación esta lesión pulmonar?
de adultos A) Biopsia guiada por tomografía
C) Los medicamentos no se recomiendan en B) Tomografía de tórax
la depresión infantil C) Broncoscopia
D) Los niños deprimidos rara vez D) Evaluar con radiografía de tórax previa
experimentan síntomas psicóticos E) Obtener una tomografía de cuerpo
E) Una historia familiar de trastornos completo
afectivos es común en niños con depresión
80) Un hombre de 63 años acude quejándose
78) Una mujer de 45 años acude a consulta de fatigabilidad fácil. Tiene antecedente de
por primera vez. Su última evaluación fue hipertensión tratada con lisinopril. En los
hace 6 años. Ella niega dolor torácico, disnea últimos 3 meses, ha tenido dolor en su
o palpitaciones. Niega ingerir medicamentos rodilla derecha debido a osteoartritis y ha
y problemas médicos. Al examen físico, tomado naproxeno para el alivio. Su presión
arterial es 123/78mmHg y su frecuencia significativo por soplo sistólico mejor
cardiaca de 78/min. Su rodilla derecha está escuchado en el ápex. Sus pulmones son
levemente deformada debido a claros, el examen abdominal no tiene
sobrecrecimiento óseo pero no está hallazgos anormales. Los estudios revelan
hinchada ni dolorosa. El resto del examen hemoglobina de 6.0 g/dl. El frotis de sangre
físico es normal. Los resultados de periférica con tinción de Wright muestra
laboratorio son los siguientes: cuerpos de Howell – Jolly y ausencia de
Hemoglobina: 8.6mg/dl reticulocitos. ¿Cuál de los siguientes es el
Volumen corpuscular medio: 72fl patógeno más probable?
Glóbulos blancos: 7,200/mm3 A) Virus Influenza
Plaquetas: 165,000/mm3 B) Virus Parainfluenza
Creatinina: 1.2 mg/dl C) Parvovirus
Velocidad de eritrosedimentación: 15 mm/hr D) Salmonella
¿Cuál de los siguientes cambios son los que E) Streptococcus pneumoniae
usted esperaría tuviera este paciente?
83) Un hombre de 45 años de edad que
recientemente inmigro de China, acude al
médico por disnea, fatiga y distensión
abdominal de 2 meses. No tiene
antecedentes médicos y no toma
medicamentos. El paciente trabajó como
granjero toda su vida. Su temperatura es
81) Una mujer de 45 años de edad con una 36.7°C, presión de 110/60 mmHg, pulso
enfermedad mental crónica parece estar 80/min y respiración de 16/min. Al examen
constantemente masticando. Su lengua se físico hay edema pedio, aumento del
mueve dentro y fuera de su boca y de vez en perímetro abdominal con líquido libre,
cuando se golpea sus labios. Ella también aumento dela presión venosa yugular sin
hace una mueca, frunce el ceño y parpadea declive respiratorio. A la auscultación
en exceso. Estos movimientos anormales se cardiaca hay disminución de los ruidos
ven, característicamente, en cardiacos y un acentuado sonido directo
A) Sindrome de Tourette después del segundo sonido en la diástole
B) Acatisia temprana. La radiografía muestra un anillo
C) Discinesia tardía de calcificación alrededor del corazón y las
D) Enfermedad de Parkinson curvas de presión yugular venosa muestran X
E) Enfermedad de Wilson prominente y Y descendentes. ¿Cuál de los
siguientes es la causa más probable de los
82) Una mujer de raza negra de 19 años de síntomas de este paciente?
edad con anemia falciforme ha presentado A) Cor pulmonare
palpitaciones y mareos por 3 días. Ella B) Pneumoconiosis
trabajo medio tiempo en una guardería. Su C) Psittacosis
temperatura es 38.1°C, pulso de 110/min y D) Infección por Trypanosoma cruzi
respiración es 18/min. El examen cardiaco es E) Tuberculosis
84) Cada cuatro o cinco semanas, una mujer de gonorrea durante su servicio militar y una
de 35 años de edad, por lo general en buen apendicectomía a la edad de 36 años. El
funcionamiento y temperamento, examen físico revela un hombre delgado, de
experimenta unos días de irritabilidad, llanto mediana edad con el habla apresurada. Su
inexplicables en estos días, también se siente examen pupilar es normal, excepto que las
cansada e hinchadoa y come grandes pupilas son reactivos a la convergencia, pero
cantidades de dulces. ¿Cuál es el diagnóstico no a la luz. La infección con cuál de los
más apropiado? siguientes organismos podría ser la causa
A) Ciclotimia más probable de sus síntomas?
B) Trastorno límite de la personalidad (A) Chlamydia trachomatis
C) Trastorno de identidad disociativo (B) Virus Herpes simple
D) Trastorno disfórico premenstrual (C) Neisseria gonorrhoeae
E) Trastorno depresivo menor (D) Toxoplasma
(E) Treponema pallidum
85) Una mujer joven es dejada en la puerta
de la sala de emergencias local por dos 87) Una mujer de 19 años de edad se
hombres que se alejan de inmediato en presenta a su médico de atención primaria
carro. Ella está agitada y ansiosa y por exceso de hematomas en las piernas
contínaumente se lava y restriega los brazos durante los últimos 3 días. Ella niega lesión.
y las piernas "para deshacerse de los Fue tratada por faringitis por estreptococo
insectos." Ella se agarra en el pecho, hace 10 días y ha completado recientemente
gimiendo de dolor. Sus pupilas están la terapia con antibióticos. Refiere un poco
dilatadas y su presión arterial es elevada. La de dolor abdominal tipo cólico, pero está en
sustancia más probable que ha consumido sus días premenstrual y dice que el dolor es
es: similar a su calambres habituales. Tomó
A) Alcohol ibuprofeno para el dolor, con un buen alivio.
B) Heroína Tiene un novio, pero no es sexualmente
C) Alprazolam activa. Ella niega historia previa de
D) Diluyente de pintura moretones o sangrado fácil. La inspección de
E) Crack las piernas revela dolor difuso, parches
eritematosos, endurecidos y nódulos en los
86) Un veterano de 45 años de edad, es aspectos anteriores de su tibias
visto en la clínica por su pobre control de los bilateralmente. ¿Cuál de las siguientes es la
impulsos. En el último mes, ha invertido causa más probable de sus síntomas?
todos sus ahorros en empresas de negocios y A) Reacción de hipersensibilidad tardía
actualmente está distanciado de su esposa. después de la infección por estreptococo
Él admite una menor necesidad de sueño, un B) Violencia doméstica
nuevo sentido de grandiosidad, y aumento C) Púrpura de Henoch – Schönlein
de la energía dedicada a la investigación de D) Púrpura trombocitopénica idiopática
proyectos para hacer dinero. Él no tiene E) Sífilis secundaria
antecedentes psiquiátricos previos, y su
única historia clínica incluye el tratamiento
88) Un hombre de 25 años tiene máculas Hemoglobina de 8.6 g/dl
pigmentadas en las palmas de la manos, Hematocrito: 27%
plantas de los pies, y mucosa oral. Presenta Plaquetas: 164,000/mm3
dolor abdominal y anemia. El diagnóstico La morfología de los eritrocitos es
más probable es: significativo por la apariencia de pila de
A) Síndrome de Albright monedas. La velocidad de
B) Síndrome de Cushing eritrosedimentación es de 55mm/hr.
C) Síndrome de Peutz-Jegher Proteínas de Bence Jones se han identificado
D) Incontinencia pigmentaria en orina. ¿Cuál de los siguientes es lo
esperado a la evaluación de la médula ósea
89) Una mujer de 23 años es llevada a del paciente?
urgencias por dificultad respiratoria. Sus A) Sobreproliferación de células plasmáticas
síntomas iniciaron hace 1 hora cuando ella B) Aumento de la celularidad de médula ósea
fue picada por una abeja en el parque. Ella con hiperplasia megacariocítica
tiene historia de asma leve intermitente. C) Hipoplasia grasa de la médula sin células
Entre los medicamentos que toma incluyen anormales.
anticonceptivos orales, multivitaminas y el D) Médula ósea normocelular
albuterol. No tiene antecedentes de E) Médula ósea fibrótica e hipocelular.
alergias. Su temperatura es de 37.1°C,
presión arterial de 82/50 mmHg, su pulso de 91) . Una mujer previamente sana de 25 años
108/min y respiración de 26/min. La de edad, sin antecedentes médicos se
evaluación muestra una mujer consciente, en presenta al servicio de urgencias quejándose
dificultad respiratoria con sibilancias de 4 horas de dolor torácico, dificultad para
audibles. Su piel está caliente al tacto. ¿Cuál respirar, disnea de esfuerzo, y "no sentirse
de los siguientes es el mejor próximo paso bien" durante los últimos días. Ella niega el
apropiado en el manejo de esta paciente? consumo de drogas ilícitas o alcoholismo. Los
A) Nebulización continua con albuterol signos vitales muestran presión arterial de
B) Epinefrina intramuscular 92/60, Frecuencia cardiaca del 135 por
C) Corticosteroides intravenosos minuto, respiratoria de 30/min, y la
D) Difrenhidramina intravenosa temperatura de 38.6°C. El
E) Epinefrina subcutánea electrocardiograma muestra ritmo sinusal
normal con cambios inespecíficos de la onda
90) Un hombre de 62 años acude a consulta T. La radiografía de tórax es normal. Además
por queja de aumento de la fatiga y debilidad de oxígeno, cuál de los siguientes representa
en los últimos 4 meses. También describe un el mejor paso a seguir?
dolor sordo en su espalda y brazos, el cual (A) Aspirina, nitroglicerina, verificar los
empeora al caminar. Niega molestias de niveles de troponina y mioglobina
parestesias o adormecimientos. Al examen (B) Aspirina, bloqueador beta, niveles TSH
físico impresiona pálido. Hay dolor alrededor (C) Líquidos intravenosos, analgésicos,
de la columna lumbar. El hemograma revela: ecocardiograma de urgencia
Glóbulos blancos 8,600/mm3 (con diferencial (D) Aspirina, nitroglicerina, centelleo de
normal) ventilación perfusión urgente
(E) Fluidos por vía intravenosa , lorazepam, el sangrado es interno. El paciente grita que
antiácidos no va a tolerar el maltrato por más tiempo y
trata de ponerse de pie, momento en el que
92) Un hombre de 44 años de edad con una comienza a vomitar. La sangre se derrama de
larga historia de consumo de alcohol y su boca, y el paciente dice: "Aquí vamos de
presunta gastritis se presenta a emergencias nuevo." El equipo de la ambulancia le dice al
dolor abdominal de inicio repentino y médico que había también una gran cantidad
vómitos. Los signos vitales son presión de vómito en el bar donde fue encontrado. El
arterial (PA) de 110/60 mmHg, frecuencia médico es capaz de controlar al paciente y
cardíaca de 110 latidos por minuto, obtener sus signos vitales. Su presión
temperatura de 38°C, y frecuencia sanguínea está 118/78 mm Hg, el pulso es de
respiratoria de 30 respiraciones por minuto. 98 / min, la frecuencia respiratoria es 22 /
El examen físico revela diaforesis y dolor min, y la temperatura es 37,2 ° C. El médico
epigástrico con defensa leve. Laboratorios no puede obtener una historia del paciente o
revelan un conteo de glóbulos blancos de contactar con parientes o amigos. No se
30.000/mm3 y un nivel de amilasa de 2.000 observan signos de trauma obvio. ¿Cuál es el
UI/L. La radiografía de tórax muestra una mejor paso siguiente en el diagnóstico?
pequeña cantidad de aire libre bajo el (A) Esofagografía
diafragma. ¿Cuál es el diagnóstico más (B) Electrocardiograma
probable? (C) La endoscopia
(A) Pancreatitis aguda con síndrome de (D) Manometría esofágica
Mallory-Weiss asociada (E) Rayos X de tórax
(B) Pancreatitis aguda con síndrome de
Boerhaave asociada 94) Masculino de 60-años de edad, ha tenido
(C) Pancreatitis aguda asociada con tos crónica con producción de esputo claro
destrucción enzimática de la pared intestinal durante más de 5 años. Ha fumado un
(D) Pancreatitis aguda secundaria a paquete de cigarrillos por día durante 20
perforación úlcera duodenal anterior años, y continúa haciéndolo. Radiografía de
(E) Pancreatitis aguda secundaria a tórax muestra infiltrados sin hiperinflación.
perforación de úlcera duodenal posterior La gasometría arterial muestran un pH de
7,38, PCO2 de 40 mmHg, y PO2 de 65 mmHg.
93) Un hombre blanco de 45 años de edad La espirometría muestra un FEV1 / FVC del
con historia desconocida es llevado a 45% sin respuesta broncodilatadora. ¿Cuál
urgencias en ambulancia. Él está agitando de las siguientes es la modalidad de
violentamente sus manos, tratando de tratamiento más importante para este
golpear “los palos voladores que están a su paciente?
alrededor”. Él está muy agitada y con fuerte (A) Corticosteroides orales
olor a alcohol. El equipo de la ambulancia (B) Oxígeno en el hogar
informa al médico de que el paciente se (C) Antibióticos de amplio espectro
encontró sangrando por la boca afuera de un (D) Programa para dejar de fumar
bar. No pudieron encontrar ninguna (E) Teofilina oral
laceración en la boca o los labios y creen que
95) Una mujer de 60 años de edad, se queja glóbulos blancos es 14.000. ¿Cuál es la
por fiebre y dolor constante cuadrante probable causa del empeoramiento de su
inferior izquierdo de 2 días de duración. Ella anemia?
niega vómitos o sangrado rectal. Tiene (A) Deficiencia de ácido fólico
antecedente de hipertensión, pero es por lo (B) Leucemia mieloide aguda
demás sana. Ella nunca ha tenido dolor (C) Mielofibrosis
abdominal similar y no ha tenido cirugías (D) Tuberculosis
previas. Su única medicación habitual es (E) Infección viral
lisinopril. El examen, su presión arterial es
150/80mmHg, pulso 110/min, y la 97) . Un joven de 17 años de edad, es llevado
temperatura de 38,9°C. Los ruidos a la sala de urgencias por confusión y falta de
intestinales normales y Dolor abdominal en coordinación. Él no coopera y se niega a
el cuadrante inferior izquierdo con rebote. proporcionar más historia. El examen físico
Un hemograma revela Leucos en 28.000 revela una frecuencia respiratoria de 30 rpm;
cel/mm3, electrolitos séricos, nitrógeno de El resto de los signos vitales son normales así
urea, creatinina y pruebas de función como el examen físico general. Los valores
hepática son normales. ¿Cuál es el siguiente de laboratorio son los siguientes:
mejor paso para evaluar el problema de este Na: 135 mEq / L
paciente? K: 2,7 mEq / L
(A) Colonoscopia HCO3: 15 mEq / L
(B) Enema de bario Cl: 110 mEq / L
(C) Laparotomía exploratoria Ca: 9,7 mg / dl
(D) Ultrasonido abdominal PO4: 4,0 mg / dL
(E) Tomografía computarizada de abdomen y Gasometría arterial: PO2: 90, PCO2: 30, pH:
pelvis 7,28
Orina: pH 7,5, glucosa-negativas ¿Cuál de las
96) Un hombre de 78 años de edad, se queja siguientes es la causa más probable de la
del aumento de la fatiga y el dolor en los enfermedad ácido-base?
huesos, especialmente alrededor de las (A) Pérdida Gastrointestinal debido a la
rodillas y tobillos. Ha sido anémico durante diarrea
varios años, con hemoglobina de 9 a 10 g / dl (B) Acidosis tubular renal proximal
y VCM de 102. Sus leucocitos y plaquetas han (C) Trastorno del sistema renina-
sido normales; No ha tenido linfadenopatía o angiotensina
esplenomegalia. No ha tenido respuesta a las (D) Acidosis tubular renal distal
pruebas terapéuticas de hierro y vitamina (E) Acidosis respiratoria
B12, pero ha estado estable Preguntas 98 a 102: Para cada situación
sintomáticamente hasta el mes pasado. El clínica, seleccione la combinación de
examen revela palidez y la punta del bazo es gasometría arterial y valores de pH con el
palpable en el margen costal izquierdo. El que es más probable que esté asociado. Cada
hemograma revela hemoglobina de 8,2 g / opción de letras puede utilizar una vez, más
dl, pero por primera vez su recuento de de una vez, o nada en absoluto.
plaquetas es bajo (15.000); el recuento de (A) pH 7.50, PO2 75, PCO2 28
(B) pH 7.14, PO2 78, PCO2 95 Después de la administración intravenosa de
(C) pH 7.06, PO2 36, PCO2 95 líquidos, los vómitos se resuelven, y es capaz
(D) pH 7.06, PO2 108, PCO2 13 de tolerar la hidratación oral. ¿Cuál de las
(E) pH 7.37, PO2 48, PCO2 54 siguientes sería indicación para que el
paciente debe ser ingresado en el hospital?
98) Un conductor de autobús, femenina, (A) ALT elevada, que indica daño
obesa de 30 años de edad, desarrolla hepatocelular
repentinamente un dolor torácico tipo (B) Hiperglucemia
pleurítico de lado izquierdo, hemoptisis y (C) Vómitos sintomático
disnea. (D) Prolongación del tiempo de protrombina
(E) Edad mayor de 30 años
99) Un gran fumador de 60 años de edad,
tiene bronquitis crónica severa, edema 104) Una mujer hispana de 44 años de edad
periférico, y cianosis. acude a la clínica para un chequeo general,
debido a la preocupación por una historia
100) Un hombre de 22 años de edad, adicto familiar de diabetes y presión arterial alta. Su
a las drogas es llevado a la sala de altura es de 157.5cm, pesa 50 kg (110 lb),
emergencias por sus amigos que no pudieron tiene una circunferencia de cintura de 33
despertarlo. pulgadas (85 cm), la presión arterial 138/88
mmHg. Los laboratorio revelan: glucosa en
101) Un hombre de 62 años de edad con ayunas de 120 mg / dL. Colesterol total de
bronquitis crónica desarrolla dolor en el 240 mg / dl, HDL 38 mg / dl y triglicéridos
pecho y se le da oxígeno a través de la 420 mg / dl; LDL no se puede calcular. Ella no
máscara en la ambulancia de camino al fuma, utiliza alcohol, ni toma medicamento
hospital. En la sala de urgencias su estado de alguno. ¿Cuál de los siguientes es correcta
conciencia es ahora aletargado. con respecto a la identificación del síndrome
metabólico en este paciente?
102) Un hombre de 20 años de edad con (A) El síndrome metabólico no está presente
diabetes mellitus llega a la sala de urgencias en este caso debido a la ausencia de
con dolor abdominal difuso, taquipnea y obesidad abdominal.
fiebre. (B) El síndrome metabólico no está presente
debido a que la presión arterial no está lo
103) Un hombre de 35 años de edad, suficientemente elevada para ser un factor
expuesto a la hepatitis A se presenta con de riesgo.
ictericia de nueva aparición y vómitos. Los (C) El síndrome metabólico no está presente
signos vitales son normales, y el examen debido a que la glucosa no está
físico mostró hepatomegalia leve, con dolor suficientemente elevada para ser un factor
en el cuadrante superior derecho. Los de riesgo.
estudios de laboratorio muestran un ALT de (D) El síndrome metabólico está presente en
300 U / l; bilirrubina total de 9 mg / dl; base a los factores de riesgos dados.
tiempo de protrombina de 16,0 seg (normal (E) El síndrome metabólico no se puede
10.0-12.0 s), y la glucosa de 75 mg / dL.
identificar hasta que se determine el nivel de aumentando en un 98% con 4 l de oxígeno a
LDL través de cánula nasal. Al examen físico es
importante la presencia de crepitantes en el
105) Una mujer de 42 años de edad con campo inferior del pulmón derecho y un
antecedentes familiares de enfermedad signo de Homans negativo bilateralmente. La
renal poliquística del adulto se presenta al radiografía de tórax parece normal, dímero D
servicio de urgencias quejándose de gran está elevado y el ECG muestra taquicardia
cantidad de sangre en su orina, 3 días de sinusal, desviación del eje derecho, onda S
fiebre y dolor abdominal del lado derecho. en la derivación I, onda Q profunda en D III, y
Revela que ella ha tenido también disuria y una onda T invertida en D III. ¿Cuál de los
frecuencia urinaria. Ha tenido síntomas siguientes es el paso más apropiado en el
similares dos veces en el año pasado y fue diagnóstico?
tratado por una infección del tracto urinario. (A) Gasometría arterial
Su temperatura es de 38,6 ° C, pulso es de 82 (B) Ecografía Doppler de las extremidades
/ min, frecuencia respiratoria es 18 / min, y inferiores
presión arterial es 160/100 mm Hg. El (C) Resonancia Magnética
urinálisis muestra glóbulos blancos y rojos, (D) Angiografía pulmonar
así como proteínas 1+ y esterasa leucocitaria (E) Centelleo de ventilación / perfusión
positiva. ¿Cuál de los siguientes es más
probable que sea una iatrogenia en este 107) Una mujer de 60 años de edad con
paciente? depresión y un mal control de la diabetes
(A) Ampicilina mellitus tipo 2 se queja de vómitos
(B) Inhibidor de la enzima convertidora de la episódicos en los últimos tres meses. Ella
angiotensina tiene náuseas constantes y saciedad
(C) Gentamicina temprana. Vomita una o dos veces casi todos
(D) Dieta baja en proteínas los días. Además, informa de varios meses de
(E) Drenaje percutáneo del quiste malestar (tipo dolor) abdominal leve que se
localiza en la parte superior del abdomen y
106) Una mujer de 27 años de edad tiene 7 que a veces la despierta por la noche. Ella ha
meses de embarazo de su primer hijo. Su perdido 5 libras de peso. Su diabetes mal
embarazo ha sido normal hasta la fecha. controlada (hemoglobina glicosilada
Refiere en urgencias inicio súbito, de dolor recientemente fue de 9,5). Los
en el pecho del lado derecho que se medicamentos actuales son glibenclamida,
exacerba con la respiración profunda, falta metformina y amitriptilina. Su examen físico
de aliento, que comenzó hace poco más de 1 es normal, excepto por la distensión
hora. Niega dolor en las piernas, y dice que el abdominal leve y la evidencia de una
edema evidente no ha cambiado desde el neuropatía sensorial periférica. Hemograma
sexto mes de embarazo. Su temperatura es completo, electrolitos séricos, Nitrógeno de
de 37,9 ° C, presión arterial es 130/87 mm urea, creatinina, y pruebas de función
Hg, el pulso es de 107 / min, regular, hepática son normales. Ultrasonido de la
respiratoria es de 24 / min, y la saturación de vesícula biliar es negativo por cálculos
oxígeno es del 90% a aire ambiente, biliares. Serie esofagogastroduodenal y
tomografía computarizada del abdomen son Calcio: 11,5 mg / dL
normales. ¿Cuál es el mejor paso siguiente Radiografía de tórax: infiltrado en el lóbulo
en la evaluación de los síntomas de este inferior izquierdo
paciente? Recuento de reticulocitos: 1%
(A) Esofagograma con bario El diagnóstico definitivo se hace mejor
(B) Centellografía del vaciamiento gástrico mediante cuál de las siguientes opciones?
(C) Colonoscopia (A) Proteínas en orina de 24 horas
(D) Biopsia hepática (B) Radiografías óseas
(E) Biopsia de intestino delgado (C) Biopsia renal
(D) fenómeno de pila de monedas
108) Una niña de 5 años de edad con (Rouleaux) en frotis de sangre
leucemia linfoide aguda ha estado (E) Células plasmáticas > 30% en médula
hospitalizada en el servicio de Hematología y ósea
oncología pediátrica durante más de 2
meses. Un médico ha sido asignado a 110) Una mujer de 53 años de edad se
cuidarla desde su primer día. Durante este presenta al servicio de urgencias con una
tiempo, la joven paciente ha tenido muchas lesión menor y se encontró que tenía una
infecciones, incluyendo sepis por Candida. presión arterial de 150/102, posiblemente
Durante las últimos 2 semanas, su principal elevada como consecuencia del dolor. En el
problema ha sido la hipopotasemia seguimiento en su consultorio su presión
(hipokalemia) persistente, a pesar de la arterial en dos ocasiones es de 142/94
reposición diaria. ¿Qué valor laboratorio mmHg, a pesar de buenos hábitos
debe al médico comprobar en este paciente, alimentarios y de ejercicio regular. Su
y es que se espera que sea alto o bajo? historia y examen físico son normales sólo
(A) Ca2 +, alta con el antecedente de una histerectomía.
(B) Ca2 +, baja Evaluación los laboratorios basales no revela
(C) Mg2 +, alta anomalías significativas. Sobre la base de las
(D) Mg2 +, baja recomendaciones del JNC 7 y 8 (El Séptimo y
(E) Na +, alta Octavo Informe del Comité Nacional
Conjunto sobre Prevención, Detección,
109) Masculino de 70 años de edad, con Evaluación y Tratamiento de la Hipertensión
queja de 2 meses de dolor de espalda y Arterial) cuál de los siguientes es información
fatiga. Ha desarrollado fiebre con producción precisa para darle a la paciente?
de esputo purulento. En el examen físico, (A) En el grupo etario de > 50 años, la presión
tiene dolor durante varias vértebras y arterial diastólica elevada se convierte en un
estertores en la base izquierda. Los factor de riesgo cardiovascular más
resultados de laboratorio son los siguientes: importante que la presión arterial sistólica
Hemoglobina: 7 g / dl elevada.
VCM: 89 fL (normal 86-98) (B) La nueva clasificación de la
Glóbulos blancos 12.000 / ml prehipertensión se ajusta a sus últimas
Nitrógeno de urea: 44 mg / dl lecturas de la PA; continuar un seguimiento
Creatinina: 3,2 mg / dL cercano.
(C) Los diuréticos tiazídicos sería una buena 112) Una mujer de 62 años de edad se
elección inicial para ella. presenta a su ginecólogo por sangrado
(D) Iniciar el tratamiento con dos vaginal. Ella tuvo su último ciclo menstrual a
antihipertensivos sería preferible basado en los 45 años y no ha tenido ningún episodio
su actual nivel de presión arterial. de sangrado vaginal desde entonces. Ella
(E) La terapia de reemplazo de estrógenos afirma que notó por primera vez algunas
podría ser útil en retrasar su necesidad de manchas de sangre en su ropa interior hace
antihipertensivos. aproximadamente 3 meses; y sigue
manchado "de vez en cuando", pero no ha
111) Una mujer de 69 años de edad se aumentado en cantidad. Es sexualmente
presenta a su médico de atención primaria activo con su marido de 40 años, pero no ha
quejándose de fatiga y malestar en los notado relación entre la actividad sexual y el
hombros y las caderas. Ella dice que en los sangrado. Sufre de hipertensión por la que
últimos 2 meses, ha perdido 4,5 kg (10 libras) toma "una píldora para el agua", pero por lo
y se siente cansada todo el tiempo. Todo su demás es sana. Un examen con espéculo
cuerpo se siente rígido en la mañana durante revela una cúpula vaginal atrófica, sin
al menos media hora. Tiene problemas para evidencia de sangre. El cuello del útero es
vestirse, ya que es difícil ponerse una camisa pequeño y no friable. Un examen pélvico
o tirar hacia arriba de sus medias. En el revela un útero pequeño, no doloroso, y sus
examen físico, hay reducido rango de ovarios no son palpables. ¿Cuál de los
movimiento activo en los hombros y caderas. siguientes es el siguiente paso más
No tiene dolor muscular. La fuerza es 5/5 en apropiado?
las cuatro extremidades. Sin focalización. Los (A) Ecografía abdominal (Ultrasonido)
hallazgos de laboratorio relevantes son los (B) Biopsia endometrial
siguientes: (C) Examen de seguimiento de 6 meses
Glóbulos blancos 8600 / mm3, (D) Niveles séricos de la hormona
Hemoglobina 10,4 g / dl, Plaquetas de luteinizante (LH) y de la hormona
160.000 / mm3 estimulante del folículo (FSH)
Velocidad de eritrosedimentación de 120 (E) Recetar crema de testosterona
mm / h,
Creatina quinasa 52 U / l 113) Una mujer de 32 años de edad se
Un ciclo corto de prednisona conduce a la presenta al servicio de urgencias con edema
resolución casi completa de los síntomas. y dolor de la extremidad inferior derecha que
¿Cuál es el diagnóstico más probable en este inició después de un viaje en auto de 8 horas.
paciente? En el ultrasonido doppler de urgencias se
(A) Fibromialgia observó una trombosis venosa profunda. La
(B) Osteoartritis paciente no tiene antecedentes de trombosis
(C) Polimialgia reumática venosa profunda o embolia pulmonar, ha
(D) Polimiositis estado tomando píldoras anticonceptivas
(E) Artritis reumatoide orales durante los últimos 5 años y con
buena adherencia. En sus antecedentes
familiares refiere que su abuela materna,
madre, y hermana han tenido trombosis títulos de anti- ADN de doble cadena
venosa profunda recurrente. Su temperatura marcadamente elevados. El reumatólogo
es de 36,2 ° C, su presión arterial está 112/78 descubrió que tenía rastros de sangre en la
mm Hg, su frecuencia cardíaca es de 86 / orina y proteinuria. La biopsia renal estaba
min, y su frecuencia respiratoria es de 14 / programada, y ella inició en un curso de 2
min. No hay evidencia clínica indicativa de meses de prednisona. En el transcurso de su
embolia pulmonar. ¿Cuál es la causa más terapia con la medicación cuál es el efecto
probable de su trombosis venosa profunda? adverso que esta paciente está más
(A) Deficiencia de Antitrombina propensa a experimentar?
(B) Mutación del gen del factor V de (A) Pérdida de cabello
coagulación (B) Hipertensión arterial
(C) Exceso de proteína C (C) Hipoglucemia
(D) Deficiencia de proteína S (D) Leucopenia
(E) Mutación del gen de protrombina (E) Pérdida de Peso

114) Una mujer de 30 años de edad se 116) Un bebe de 4 años de edad es llevado a
presenta a un médico de atención primaria urgencias con una fiebre de 40,6 ° C. Hace 5
para control de salud. Ella informa episodios días fue visto por su pediatra por inyección
recurrentes de neumonía, bronquitis y otitis conjuntival bilateral y fiebre de 39 ° C. Ha
en los últimos 4 años. A pesar de que sus sido un niño sano y sus vacunas estén al día.
vacunas estaban al día, desarrolló tétanos Su frecuencia respiratoria es de 22 / min,
después de una laceración del pie el año frecuencia cardíaca 130 / min, y presión
pasado. Sus ganglios linfáticos y amígdalas arterial en 105/63 mm Hg. Presenta
están agrandadas. Las pruebas de laboratorio inyección no exudativa de su conjuntiva
revelan bajos niveles de IgG, IgA e IgM. bulbar bilateral. Sus labios son eritematosos,
Después de ser remitido a un hematólogo, secos y agrietados, y su lengua es de color
que descarta otras causas adquiridas y rojo brillante. Tiene una erupción no
genéticas de la hipogammaglobulinemia, se vesicular polimorfa en su tronco. No tiene
diagnostica con inmunodeficiencia variable adenopatías cervical, occipital, o inguinales.
común. Esta mujer tiene riesgo más elevado Tiene un S1 y S2 normal, sin soplos. Los
de desarrollar cuál de las siguientes ruidos intestinales normales y un abdomen
condiciones? distendido no doloroso, y sin organomegalia.
(A) Enfermedades cardiovasculares Sus manos y pies están edematosos y sus
(B) Linfoma palmas y plantas son eritematosas. ¿Cuál de
(C) Aborto espontáneo los siguientes es el diagnóstico más
(D) Enfermedad renal probable?
(E) Autoinfarto esplénico (A) Infección por Adenovirus
(B) Enfermedad de Kawasaki
115) Una mujer de 23 años de edad ha sido (C) Sarampión
recientemente diagnosticada con lupus (D) Infección por Staphylococcus aureus
eritematoso sistémico. En el momento del (E) Síndrome de Stevens-Johnson
diagnóstico, tenía un VES elevado y los
117) Una mujer de 48 años de edad con
antecedentes de sarcoidosis llega al servicio 119) Un hombre de 32 años es llevado a
de urgencias por síncope. Ella afirma urgencias por los paramédicos luego de
anorexia, náuseas, vómitos y dolor encontrarlos calle abajo aparentemente
abdominal. Dejó de ir a trabajar porque está agitado y delirando. Durante el traslado al
demasiado cansada. El examen físico revela hospital, el paciente se vuelve diaforétigo y
lesiones hiperpigmentadas y alopecia. trémulo y se detecta hpertenso en 163/100
Hallazgos de laboratorio iniciales muestran mmHg, taquicárdico y con pulso de 102/min,
un nivel de glucosa de 50 mg / dL y un nivel febril 39°C. Al examen se le observan
de potasio de 5,4 mEq / L. ¿Cuál es el dilatación pupilar y ulceraciones en la
diagnóstico más probable? mucosa del septo nasal con residuos de un
(A) Síndrome de de Conn polvo blanco a lo largo del ala nasal. ¿Cuál es
(B) Diabetes insípida nefrogénica la razón por la cual se evita utilizar
(C) Hipertiroidismo betabloqueadores en este paciente?
(D) Insuficiencia suprarrenal (A) Aumenta riesgo de vasoespasmo tardío
(E) Depresión (B) Riesgo de agravar agudamente la
hipertensión a través de la vasoconstricción
118) Un hombre VIH positivo de 47 años de (C) Riesgo de causar hipotensión aguda
edad, es llevado a la sala de urgencias por (D) Riesgo de causar disnea
debilidad. El paciente tiene nefropatía VIH e (E) riesgo de arritmia ventricular
insuficiencia suprarrenal. Toma Trimetropin
– sulfametoxazol para profilaxis de PCP y 120) Una niña de 18 años de edad, presenta
está recibiendo tratamiento antirretroviral a su obstetra / ginecólogo por disuria y
triple. Recientemente inició el aumento frecuencia urinaria. El cultivo de
espironolactona por ascitis debido a orina muestra Escherichia coli, y el paciente
enfermedad hepática alcohólica. El examen inicia un curso de 3 días de ciprofloxacina.
físico revela signos vitales normales, pero sus ¿Cuál de las siguientes reacciones adversas
músculos tienen debilidad difusa. Se ocurre más comúnmente con esta clase de
observaron extrasístoles frecuentes. Hay medicamentos?
ascitis moderada y edema periférico 1+. Los (A) Reacciones alérgicas y erupciones
estudios de laboratorio muestran una cutáneas
creatinina sérica de 2,5mg/dl con un valor de (B) Malestar gastrointestinal con anorexia,
potasio de 7,3 mEq / L. El ECG muestra náuseas y vómitos
Ondas T picudas y QRS de duración de 0,14s. (C) Dolor de cabeza leve y mareos
¿Cuál es el tratamiento inmediato más (D) Prolongación del intervalo QT en el ECG
importante? (E) Tendinitis y rotura del tendón
(A) Sulfonato de poliestireno sódico
(Kayexalate) 121) Un varón de 54 años de edad, va a tu
(B) Hemodiálisis aguda consulta para seguimiento por hipertensión y
(C) Solución salina normal IV trastornos convulsivos que están bien
(D) Gluconato de calcio IV controlados. Se estableció como paciente
(E) Furosemida 80 mg IV stat nuevo hace 2 meses y regresa para su
segunda. En su primera visita, admitió 123) A los 26 años de edad, una joven que
antecedente de 35 años de fumar 2 paquetes practica jogging, se presenta a su médico de
de cigarrillos por día. En ese momento le atención primaria quejándose de dolor en la
indicó que no estaba interesado en dejar de parte inferior de su pierna izquierda cuando
fumar, y lo notó irritado cuando se le sugirió ejercita. Ella afirma que cuando no está
que dejara de fumar. Hoy su presión arterial corriendo, es libre de dolor, pero cuando se
está bien controlada y no hay nuevos trota de 3 millas, comienza a notar el dolor y
problemas médicos. Con respecto a la la rigidez en la pierna inferior izquierda.
discusión de la cesación del tabaquismo Afirma que a menudo tiene entumecimiento
durante la visita de hoy, ¿cuál es el mejor y hormigueo concurrente en la parte
paso siguiente? superior de su pie durante estos episodios de
(A) No hable de la cesación del hábito de dolor. Cuando cesa la actividad vigorosa,
fumar, ya que probablemente le molestará todos los síntomas desaparecen poco a poco
de nuevo. en el transcurso de media hora. El examen
(B) No hable de la cesación del hábito de físico aporta poco, demuestra una
fumar, porque no hay ningún beneficio real extremidad inferior izquierda intacta en el
para la cesación del hábito de fumar después componente neurovascular con
de haber fumado tanto tiempo. compartimentos flexibles, y no hay áreas
(C) Preguntarle si todavía está fumando y ,sí focales de dolor. Las radiografías simples de
es así, aconséjele de fumar; evalúe su la rodilla izquierda, la tibia y el peroné son
disposición a hacerlo (dejar de fumar). igualmente dentro de lo considerado normal.
(D) Recomendar bupropión. ¿Cuál es el diagnóstico más probable?
(E) Recomendar usar el cigarrillo sin humo. (A) Síndrome compartimental agudo
(B) Síndrome compartimental de esfuerzo
122) Una mujer blanca de 55 años de edad, (C) Osteoartritis de rodilla izquierda
ha tenido episodios recurrentes de (D) Síndrome doloroso patelofemoral del
pancreatitis inducida por el alcohol. A pesar lado izquierdo
de la abstinencia, la paciente desarrolla dolor (E) Fractura por estrés de la tibia izquierda
abdominal postprandial, distensión
abdominal, pérdida de peso a pesar del buen Preguntas 124 a 126
apetito, heces malolientes y voluminosas. Seleccionar el manejo más eficaz para cada
Evaluación radiológica de riñones uréter y paciente con enfermedad pericárdica.
vejiga muestra calcificaciones pancreáticas. (A) Antibióticos
En este paciente, Usted espera encontrar (B) Colocación de tubo pleural
cuál de las siguientes? (C) Diálisis Renal
(A) Diabetes mellitus (D) Corticosteroides
(B) Mala absorción de las vitaminas (E) Fármacos anti-inflamatorios no
liposolubles D y K esteroideos
(C) Sangre oculta en heces positivo (F) Pericardiocentesis urgente
(D) Signo de Courvoisier (G) Fluidos por vía intravenosa
(E) Marcada elevación de amilasa (H) Diuréticos de asa
(I) Morfina
124) Un hombre de 45 años de edad con cardiomegalia, y la ecocardiografía muestra
enfermedad renal crónica en diálisis se queja un derrame pericárdico moderado.
de dolor agudo en el pecho durante varios
días que no ha mejorado con acetaminofen. 127) Un hombre de 58 años de edad, tiene
Además, ha tenido aumento de la disnea y antecedentes de hipertensión y pregunta por
fatiga en los últimos días asociado a una tos la reducción de su riesgo de infarto de
molesta. En el examen físico, impresiona miocardio. Un perfil de lípidos muestra el
distress leve, ingurgitación yugular leve, colesterol HDL bajo en 32 mg / dl. Cuál de las
sonidos cardíacos disminuidos, y un roce siguientes es una recomendación importante
pericárdico. Sus signos vitales al triaje son en el intento de elevar el HDL?
temperatura de 37,5 ° C, pulso 85 / min, (A) Aspirina, un comprimido cada día
100/72 mm Hg de presión arterial y (B) Dieta baja en colesterol
frecuencia respiratoria de 20 / min. Su (C) Vitamina E, 400 UI cada día
saturación de O2 es del 99% a aire ambiente. (D) Suplementación con DHEA
Un ecocardiograma muestra un gran (dehidroepiandrosterona)
derrame pericárdico. (E) Ejercicio

125) Un hombre de 37 años de edad con Pregunta 128 y 129. ¿Cuál de los siguientes
antecedentes de angina y una prueba de sería el tratamiento más indicado para cada
esfuerzo positiva acude a cateterismo uno de los siguientes casos?
cardíaco. Durante el procedimiento, una de (A) Aciclovir
sus arterias coronarias es lacerada. El (B) Amantadina
paciente desarrolla taquicardia rápidamente (C) Amfotericina B
se hipotensa. El anestesiólogo avisa que su (D) Dapsone
presión sistólica cae aún más durante la (E) Doxiciclina
inspiración. Una ecocardiograma en cama (F) Foscarnet
muestra un pequeño derrame pericárdico. (G) Fluconazol
(H) Gentamicina
126) Una niña de 17 años de edad, (I) No requiere tratamiento
previamente sana acude a urgencias después (J) Penicilina
de 2 semanas de dolor en el pecho. Ella
describe el dolor como agudo, localizado en 128) Un niño de 2 años de edad que asiste a
el lado izquierdo de su pecho y se irradia a la la guardería tiene fiebre durante 3 días. La
mandíbula y el cuello. El dolor empeora fiebre por fin se calma, pero una erupción
cuando se acuesta y mejora al inclinarse maculopapular rosa se desarrolla sobre su
hacia adelante. Al examen físico hay un tronco y se extiende a los brazos y la cara. Su
suave roce pericárdico. Sus signos vitales son madre no se ha dado cuenta que el niño se
la temperatura de 37,5 ° C (99,5 ° F), pulso en rasque o le pique la erupción, pero dice que
81 / min, 139/81 mm Hg de presión arterial y el niño parece más irritable
frecuencia respiratoria de 15 / min. Su
saturación de O2 es del 100%, en aire 129) Un niño de 9 años de edad, es llevado al
ambiente. Una radiografía de tórax mostró doctor por dolor de cabeza, fiebre, vómitos,
y intenso dolor de garganta, Él también tiene despierta, de 10 a 20 minutos más tarde, él
una erupción roja que se siente como "papel se siente bien despierto y fresco.
de lija". El examen físico revela una lengua
de color rosa brillante y orofaringe Preguntas 132 a 134: Para cada paciente con
eritematosa con manchas blancas en sus un nódulo pulmonar solitario, seleccione la
amígdalas. etiología más probable.
(A) Adenocarcinoma
Pregunta 130 y 131. Relacionar cada caso (B) Aspergiloma
con el diagnóstico correcto. (C) Quiste broncogénico
A) Hipersomnia primaria (D) Tumor carcinoide pulmonar
B) Narcolepsia (E) Absceso cavitante
C) Trastorno de terror nocturno (F) Coccidioidomicosis
D) Trastorno del sueño circadiano (G) Hamartoma
E) Insomnio primario (H) Histoplasmosis
F) Trastorno del movimiento periódico de las (I) Lipoma
extremidades
G) Apnea del sueño 132) Un conductor de 55 años de edad, sin
H) Síndrome de piernas inquietas cinturón de seguridad sufre un accidente
vehicular y se queja de dolor en el lado
130) Una mujer se queja de que su marido derecho del pecho. No tiene historia médica
mueve las piernas constantemente mientras previa y niega antecedentes de tabaquismo.
duerme. Ella acaba siendo pateada varias En el examen físico hay moretones y dolor
veces en la noche. El marido no tiene sobre las costillas 8, 9 y 10 de lado derecho.
memoria de esta actividad durante la noche, Los estudios de imagen son negativos por
pero se informa que se despierta cansado fractura, pero sí revelan un solo nódulo
todas las mañanas a pesar de conseguir lo pulmonar de 1,8 cm con un patrón de
que él considera una cantidad adecuada de calcificación "palomitas o popcorn" en el
sueño (7 a 8 horas por noche) ápice pulmonar derecho, a 8 mm del
bronquio lobar superior.
131) Durante los últimos dos años, un
hombre de 28 años de edad, se ha 133) Un hombre de 43 años de edad, se
encontrado en muchas situaciones peligrosas presenta por 6 meses de tos productiva,
o incómodas debido a su hábito sudores nocturnos intermitentes y pérdida
inconveniente de caer bruscamente dormido de peso de 6,8 kg (15 libras). Niega
en medio de cualquier actividad. Una vez antecedentes de tabaquismo o viajar fuera.
chocó contra un poste porque se quedó El último conteo de CD4 + del paciente fue
dormido al volante y su esposa todavía se hace 2 semanas era de 900 / mm3. Su
burla de él por "tomar una siesta", mientras temperatura es de 37,8 ° C; sus signos vitales
que estaban teniendo relaciones sexuales. El y el examen físico son normales. La
joven informa que él comienza a soñar tan radiografía de tórax muestra una lesión de
pronto como sus ojos se cierran y cuando se 2,7 cm en el campo superior del pulmón
derecho. Tinciones y cultivos son negativos dos meses antes de la aparición de estos
para bacilos alcohol ácido resistente. síntomas ha estado cada vez más retirado,
suspicaz, desinteresados en su trabajo
134) Un hombre de 72 años de edad, va a su académico, y extrañamente indiferente
médico por tos no productiva y pérdida de
peso involuntaria de 9 kg (20 libras) en los 136) Una mujer de 35 años de edad, ha sido
últimos 4 meses. El paciente no tiene residente hospital estatal durante los últimos
antecedentes médicos, no toma 15 años. Ella pasa la mayor parte de su
medicamentos, y niega fumar. Su tiempo deambulando, murmurando en voz
temperatura es de 36,7 ° C, la presión baja para sí misma, y mirando su reflejo en
arterial es 131/84 mm Hg, el pulso es de 75 / un espejo pequeño. Ella Necesita ayuda para
min y regular, y la frecuencia respiratoria es vestirse y ducharse y que a menudo se ríe y
de 16 / min. En el examen físico hay carcajea sin motivo aparente
acropaquia. La radiografía de tórax muestra
un nódulo solitario de 1,8 cm periférica en el 137) Un hombre sin hogar de 45 años de
lóbulo inferior izquierdo. Una Tomografía edad, se niega a quedarse en un refugio
confirma la presencia de la lesión de 1,8-cm porque está convencido de que los otros
en el lóbulo inferior izquierdo que no invade asistentes son adoradores del diablo y que
la pleura visceral o bronquios lobares van a matarlo mientras duerme. Afirma que
Preguntas 135 – 139: Relacionar cada caso los ha escuchado sobre cómo harán el corte
con su diagnóstico apropiado. La opción de la garganta y el robo de sus zapatos. Los
puede seleccionarse 1 sola vez, varias veces voluntarios del refugio han oído informes de
o ninguna. temores similares durante los últimos 10
A) Desorden delirante años en este hombre.
B) Trastorno esquizoafectivo
C) Depresión psicótica 138) Un paciente hospitalizado con una
D) Esquizofrenia, tipo paranoide enfermedad mental crónica ha estado mudo
E) Intoxicación por PCP e inmóvil durante más de dos semanas. Se
F) Trastorno esquizofreniforme resiste activamente cualquier intento de ser
G) Catatonia movido. De vez en cuando tiene breves
H) Trastorno psicótico compartido períodos repentinos, no provocado de
I) Esquizofrenia, tipo desorganizado agitación y agresividad

135) Durante seis semanas, un estudiante 139) Una secretaria legal de 45 años de edad,
universitario ha sobrevivido con alimentos que lleva una vida un poco aislada, pero no
enlatados porque tiene miedo de ser aporta ningún otro hallazgo, ha escrito
envenenado por la mafia. Él está convencido cientos de cartas de amor a una famosa
de que cámaras secretas se han colocado en estrella de cine. Ella está convencida de que
su apartamento y que está siendo vigilado la esposa celosa del actor le impide declarar
constantemente, puede escuchar la voz de abiertamente su pasión por ella
un hombre que comenta en todos sus Preguntas 140-14 : Para Cada caso,
movimientos. Durante aproximadamente seleccione la prueba diagnóstica más
adecuada. (C) Angina de pecho
(A) Tinción Alcohol-Acido resistente de (D) Taponamiento Cardiaco
esputo inducido (E) fractura por compresión en la columna
(B) Pruebas metabólicas básicas vertebral
(C) Gammagrafía ósea (F) Vasoespasmo coronario
(D) Prueba de esfuerzo cardíaco (G) Espasmo de esófago
(E) Panel completo metabólica (electrolitos,
proteínas, glucosa y pruebas hepáticas y 143) Un hombre de 47 años de edad es
renales) llevado al servicio de urgencias por los
(F) Tomografía bomberos. Fue encontrado inconsciente y
(G) Ensayo de inmunoabsorción ligado a sangrando por una herida de 4 cm tórax
enzimas (ELISA) para VIH lateral izquierdo. A su ingreso, con dificultad
(H) Punción lumbar respiratoria y signos vitales con taquicardia e
(I) Biopsia muscular hipotensión, con una presión arterial de
(J) Evaluación psiquiátrica 68/43 mmHg. En la exploración física, el
(K) Ecocardiograma transtorácica paciente tiene ingurgitación yugular y los
(L) Rayos X del tórax sonidos cardíacos son difíciles de auscultar

140) Una mujer de 87 años de edad, tiene 144) Una mujer de 66 años de edad se
una historia de 3 semanas de evolución con presenta al servicio de urgencias por
fiebre y anorexia. Los cultivos de sangre, náuseas, dolor abdominal vago, y malestar
orina y esputo son negativo epigástrico. El dolor comenzó mientras ella
estaba subiendo las escaleras en la mañana y
141) Hombre de 37 años de edad con se ha incrementado gradualmente. El dolor
antecedente significativo de consumo desaparece después de 10 minutos de
excesivo de alcohol se presenta al servicio reposo, pero ella sigue preocupada. Su pulso
de urgencias con una historia de 3 semanas es de 105 / min y su presión arterial está
de evolución de fiebre y anorexia. Los 146/82 mm Hg
cultivos de sangre, orina y esputo son
negativos 145) Un hombre de 45 años de edad con
enfermedad de Parkinson tiene áreas
142) Un hombre de 87 años de edad con maculares de eritema y descamación detrás
antecedentes de un soplo diastólico de tono de las orejas y en el cuero cabelludo, las
bajo se presenta al servicio de urgencias con cejas, entrecejo, pliegues nasolabiales, y el
historia de 3 semanas de evolución de fiebre centro del pecho. ¿Cuál de los siguientes es
y anorexia. Los cultivos de sangre, orina y el diagnóstico más probable?
esputo son negativos (A) Tiña versicolor
Pregunta 143 y 144: Para cada paciente con (B) Psoriasis
dolor torácico, seleccione el diagnóstico más (C) Dermatitis seborreica
probable. (D) Dermatitis atópica
(A) Disección aórtica aguda (E) Infección por dermatofitos
(B) Infarto agudo de miocardio
146) Guías basadas en la evidencia apoyan 148) Una mujer blanca obesa de 54 años de
cuál de las siguientes evaluaciones para la edad tiene una hemoglobina A1C de 9,5 y
detección de cáncer? microalbúmina en orina elevada
(A) Radiografía de tórax en un varón de 50
años de edad, fumador de cigarrillos 149) Un hombre de 62 años de edad, tiene
(B) Mamografía en una mujer de 35 años de antecedente de infarto del miocardio y
edad con antecedentes de enfermedad angina estable crónica
fibroquística de la mama
(C) Antígeno específico prostático (PSA) en 150) Un bombero de 20 años de edad, llega a
un hombre de 80 años de edad, con un la sala de urgencias quejándose de dolor de
hermano que tiene cáncer de próstata cabeza y mareos después de ayudar a apagar
(D) Colonoscopia en un hombre de 50 años un fuego en un garaje. No se queja de
de edad, asintomático y sin antecedentes disnea, y la gasometría arterial muestra una
familiares de cáncer de colon presión parcial de oxígeno dentro de lo
(E) CA-125 en una mujer de 45 años de edad, normal. ¿Cuál de los siguientes es el mejor
con una hermana que acaba de ser primer paso en el tratamiento de este
diagnosticada con cáncer de ovario paciente?
(A) Iniciar la terapia con oxígeno.
Preguntas 147 a 149: La elección inicial de un (B) Obtener una radiografía de tórax.
antihipertensivo o la adición de otro (s) (C) Obtener un nivel de carboxihemoglobina.
posterior al régimen inicial puede depender (D) Obtener una tomografía computarizada.
de factores concomitantes. Para cada uno de (E) Evaluar por la anemia
los casos siguientes, Indique el medicamento
de elección que le daría el mejor beneficio 151) Un joven de 18 años de edad se
adicional en adición al control de la presión presenta a su consultorio. Su familia ha
arterial. Cada opción de letras puede notado una coloración azul en la nariz, orejas
utilizarse una vez, más de una vez, o nada en y dedos. Este hallazgo sólo se ha notado
absoluto. durante unas pocas semanas. ¿Cuál de los
(A) Alfabloqueador siguientes es verdadero?
(B) BetaBloqueador (A) Este paciente tiene cianosis periférica.
(C) Bloqueadores de canales del calcio (B) La hemoglobina total es importante para
(D) Inhibidor de la enzima convertidora de la determinar si es un paciente tendrá o no
angiotensina cianosis.
(E) Agonista alfa de acción central¡ (C) La cianosis central con hipoxia sugiere
(F) Diurético metahemoglobina o Sulfohemoglobinemia.
(D) La presencia de acropaquias en un
147) Un hombre afroamericano de 67 años paciente cianótico confirma una causa aguda
de edad, se queja por tener una tendencia de la cianosis.
hacia la retención urinaria. El tacto rectal (E) Se requieren grandes cantidades de
revela agrandamiento de la próstata metahemoglobina para producir cianosis.
Preguntas 152 – 154: Parear el paciente
descrito con el diagnóstico más probable.
Cada opción de letras puede utilizarse una (D) Insulinoma
vez, más de una vez, o nada en absoluto. (E) Falla Hepático
(A) Diverticulitis aguda
(B) Pancreatitis aguda 156) Cuál de las siguientes afirmaciones en
(C) Colecistitis aguda relación con el síndrome hiperglucémico no
(D) Obstrucción intestinal cetónico (SHNC) es verdadero?
(E) Síndrome del intestino irritable (A) La tasa de mortalidad del Síndrome
(F) Isquemia mesentérica hiperglicémico no cetónico (SHNC) es menor
que la de la cetoacidosis diabética (CAD)
152) Una mujer diabética de 45 años de edad (B) SHNC y CAD, son fácilmente distinguibles
se presenta con dos días de dolor abdominal (C) La mayoría de los pacientes con SHNC se
superior intenso que se irradia hacia la parte presentan con coma
posterior y se ha asociado con náuseas y (D) La acidosis metabólica excluye el
vómitos. Ella usa insulina, pero no ha sido diagnóstico
adherente durante varias semanas. Ella niega (E) Las convulsiones ocurren en hasta un 15
el consumo de alcohol. Su suero es lipémico. por ciento de los pacientes con SHNC

153) Un hombre blanco de 78 años de edad 157) Un varón de 19 años se queja de dolor
con enfermedad de la arteria coronaria se escrotal de inicio agudo. Cuál de los
presenta con varios meses de dolor siguientes procedimientos es el MENOS
abdominal generalizado postprandial que indicado?
por lo general tiene una duración de 30-60 (A) Tratarlo con cefixima o ceftriaxona y
minutos. Tiene miedo de comer y ha perdido azitromicina y darle alta
15 libras de peso (B) Centelleo con radionucleos de testículos
(C) Intentar destorción manual
154) Una mujer de 68 años de edad con (D) Urinálisis
antecedente histerectomía presenta historia (E) Ultrasonido Doppler
de 8 horas de dolor tipo cólico periumbilical.
Cada episodio de dolor dura de 3 a 5 minutos Preguntas 158 a 160: La elección de un
y luego disminuye. Durante varias horas agente antihipertensivo puede implicar el
desarrolla náuseas, vómitos y distensión evitar un efecto adverso en una condición
abdominal. Ella ha sido incapaz de eliminar comórbida. Para cada uno de los pacientes
heces o flatos durante las últimas 4 horas con hipertensión conocida, indicar el
medicamento que necesita ser evitado por
155) ¿Cuál es la causa más común de encima de todos los demás. Cada opción de
hipoglucemia en pacientes que acuden a los letras puede utilizar una vez, más de una vez,
servicios de urgencias? o nada en absoluto.
(A) Presentación de diabetes (por primera (A) Inhibidor de la enzima convertidora de la
vez) angiotensina
(B) Relacionado alcohol (B) Beta-bloqueador, no cardioselectivo
(C) Hipoglucemiantes orales (C) Bloqueante de los canales de calcio
(D) Diurético
(E) Hidralazina

158) Un hombre blanco de 40 años de edad


tiene tres episodios en los últimos 2 años de
artritis aguda debilitante que implica tobillo y
articulaciones del pie

159) Un hombre de 70 años de edad con


EPOC dejó de fumar hace 5 años, pero ahora
está comenzando a experimentar calambres
en sus músculos de la pantorrilla al caminar
una cuadra. Pulsos poplíteos y pedios
disminuidos son apreciados en el examen.

160) Una mujer soltera 24 años de edad, ha


retrasado ver a un médico durante los
últimos 3 meses, debido a la falta de seguro;
ella ha experimentado amenorrea y náuseas.

You might also like